Med 2 Flashcards

1
Q

What is the function of the hypothalamus

A
  • Key role in homeostasis = HR/BP, body temp, fluid + electrolyte balance (+thirst) app + body weight, sleep cycle, GI secretions
  • Regulates anterior pituitary gland – via releasing hormones, secretes dopamine (inhibits prolactin)
  • Synthesis hormones that are released from posterior pituitary
How well did you know this?
1
Not at all
2
3
4
5
Perfectly
2
Q

What are the two components of the pituitary gland and what do they release?

A
  • Anterior = ACTH, TSH, GH, LH, FSH, Prolactin
  • Posterior = Oxytocin (uterine contractions + breast milk ejection), ADH
How well did you know this?
1
Not at all
2
3
4
5
Perfectly
3
Q

Structure of the adrenal cortex and the hormones produced

A
  • Adrenal cortex (3main steroid hormones produced): Zona glomerulosa (-> mineralocorticoids), Zona fasciculata (-> Glucocorticoids) and Zona reticularis (->Androgens)
  • Adrenal medulla – ‘fight’ or flight response. Secretes epinephrine or norepinephrine
How well did you know this?
1
Not at all
2
3
4
5
Perfectly
4
Q

Cushings disease- the approch to treeatment

A
  • Trans sphenoidal surgery - pituitary tumours
  • Bilateral adrenalectomy - adrenal tumours eg, phaeochromocytoma
  • Ectopic ACTH - removal of tumour if not metatases, metyrapone, ketoconazole + fluconazole to reduce cortisol secretion
How well did you know this?
1
Not at all
2
3
4
5
Perfectly
5
Q

Hyperaldosteronism - what is it briefly

A

XS Aldosterone production -> increased Na+ and water reabsrption

Rare catelcholamind producing tumours eg, chromaffin cells ocllections of adrenal medulla

How well did you know this?
1
Not at all
2
3
4
5
Perfectly
6
Q

Prolactinomas and effects

A
  • High protein levels inhibit GnRH release from hypothalamus causing hypogonadism
  • Disrupt inhibitor dopamine release -> hyperprolactinaemia
  • Pituitary tumours
How well did you know this?
1
Not at all
2
3
4
5
Perfectly
7
Q

Acromegaly - one sentance description

A

Increased growth hormone release from pituitary gland

Surgery - pituitary gland tumours

How well did you know this?
1
Not at all
2
3
4
5
Perfectly
8
Q

What is hypopituitarism and the causes

A
  • Partial or complete deficiency of Ant/Post pituitary hormones. Primary or secondary to pathology. Clinical features depend on disruption to HPA.
  • Loss of GH-> LH/FSH -> TSH -> ACTH -> Prolactin
  • Pituitary apoplexy = pituitary infarction due to stalk compression
  • Sheehan’s syndrome = haemorrhage infarction of enlarged postpartum pituitary due to post partum haemorrhage
  • Causes =Compression of pituitary gland bu non secretory pituitary macroadenoma (Most common), pituitary apoplexy, sheehans syndrome, trauma, hypothalamic tumours, iatrogenic radiation, infiltrative eg, sarcoidosis).
How well did you know this?
1
Not at all
2
3
4
5
Perfectly
9
Q

Symptoms of Hypopituitarism

A
  • GH = Fatigue, muscle weakness, increase body fat
  • LH/FSH = decreased periods/pubic hair, mood changes
  • TSH = tiredness, increased weight, dry skin, cold intolerance, constipation
  • ACTH – severe tiredness dizziness, nausea + vomiting
  • Prolactin = inability to produce breast milk (Sheehan’s)
  • ADH = Increased thirst, increased urine
  • Patients usually pale to combo of mild anaemia and lack of melatonin
How well did you know this?
1
Not at all
2
3
4
5
Perfectly
10
Q

Ix and Mx of hypopituitarism

A

IX: Hormone profile testing, imaging.

Mx: Treat underlying, replace hormones.

  • Baseline Ant pituitary hormone, Serum + urine osmolarity , Dynamic tests – ITT/SST/Glucagon tests
  • Pituitary MRI
  • Visual fields ass if involvement optic chiasm
  • Replace Cortisol BEFORE Thyroxine to avoid triggering Addisonian crisis
  • TSH unreliable for monitoring – need to check FT4 + FT3
  • Need to replace Cortisol before investigating for DI
How well did you know this?
1
Not at all
2
3
4
5
Perfectly
11
Q

Adrenal insufficiency - what is it and the classifications

A

Adrenal Insufficiency: Classified as primary, secondary or tertiary and reaults from disorders that affect the adrenal cortex (eg, Addisons, Congenital adrenal hyperplasia), the anterior pituitary gland (eg, pituitary tumour or subarachnoid haemorrhage), or the hypothalamic (eg, HPA axis suppression). Acute adrenal failure is Addisons disease

•Primary adrenocortical insufficiency (Addison’s) – destruction of adrenal cortex leads to glucocorticoid (cortisol) and mineralcorticoid (aldosterone) deficiency . Causes are mostly autoimmune but others are TB, adrenal mets, lymphoma, opp infections. 30-50years. Often ass with Virtilligo.

•Secondary adrenal insufficiency – Caused by pituitary or hypothalamic disease -> decreased ACTH secretion -> adrenal failure. Most common cause iatrogenic from prolonged steroids. Commonest cause iatrogenic due to long term steroid therapy leading to suppression of pituitary adrenal axis.

How well did you know this?
1
Not at all
2
3
4
5
Perfectly
12
Q

Addison disease causes and RF

A

Autoimmune disease, TB, Adrenal mets, lymphoma, haemorrhage (WF syndrome), opportunistic infections in HIV.

•RF – Female, adrenocortical autoantibodies, adrenal haemorrhage, autoimmune disease

How well did you know this?
1
Not at all
2
3
4
5
Perfectly
13
Q

Signs and symptoms of Addison’s disease

A

•Sings/ Symptoms – Fatigue, Anorexia, Weight loss, Nausea/ vom. Hypotension, arthralgia + myalgia. Hyperpigmentation

How well did you know this?
1
Not at all
2
3
4
5
Perfectly
14
Q

Invetsigations for addisons disease

A

•– Cortisol measurements, ACTH stimulation test (short/Long synacthen tests), U&E, glucose, autoantibodies, renin/aldosterone, CXR + AXR, 21 hydroxyls auto antibodies (positive in some auto immune disease), Hypercalcaemia + anaemia.

How well did you know this?
1
Not at all
2
3
4
5
Perfectly
15
Q

Management for Addison’s disease

A
  • Replace glucocorticoid and mineralcorticoids
  • Acute – Hydrocortisone IV 100mg immediately then 200mg over 24hours either by continuous IV infusion or by 50mg iV every 6hours
  • Fluid resuscitate with 0.9% sodium chloride. Continue iV fluids for next 24-48hours
  • Convert to oral glucocorticoids once stable.
  • Non acute – systemic glucocorticoid therapy
How well did you know this?
1
Not at all
2
3
4
5
Perfectly
16
Q

How to assess and manage someone with Addisonian Crisis

A
  • Medical emergency.
  • Shock/hypotension, abdo pain, fever, hypoglycaemia, severe atigue, hyponatraemia
  • Ix = U+E, Glu, cortisol, ACTH,TFTs, Glu.
  • Mx = IVT with 0.9%saline, Give IV/IM Hydrocortisone 100mg if suspected, patient may require QDS IV Hydrocortisone util able to switch to PO tablets. Treat hypoglycaemia if present with 5% glucose infusion. Treat udnelryign cause
How well did you know this?
1
Not at all
2
3
4
5
Perfectly
17
Q

Assess and manage patients with hypovalaemic shock

A
  • Hypovalaemic shock = volume of circulatory system is inadequate for perfusion of organ tissues
  • Shock = acute circulator failure with inadequate O2 delivery. Leds to organ dysfunction, end organ damage and death
  • Diagnostic indicator = Hypotension, Tachycardia, Skin changes, Oligura
  • IX = Lactate + urine output, NEWS/OBS, Glucose, VBG/ABG, FBC, CRP, BC, ESR, WBC
  • Mx = O2 + vasoactive agents, IV access (crystalloid fluids like Hartmann’s), analgesics, treatment of underlying cause
How well did you know this?
1
Not at all
2
3
4
5
Perfectly
18
Q

Cushings SYndrome VS Disease

A
  • Cushings syndrome is adrenal excess
  • Cushings disease is a specific type of cushings when pituitary tumour causes body to make too much cortisol
How well did you know this?
1
Not at all
2
3
4
5
Perfectly
19
Q

Cushings syndrome - what is it and the 2 main classifications of cause

A
  • Pathophysiology = Caused by prolonged exposure to either endogenous or exogenous glucocorticoids.
  • ACTH Dependent – excessive ACTH from pituitary gland (Cushing’s disease) or ectopic ACTH secretion (SCLC + carcinoid tumours)
  • ACTH Independent – Adrenal cortisol excess (adrenal tumours/adenomas) or exogenous steroids.
How well did you know this?
1
Not at all
2
3
4
5
Perfectly
20
Q

What are the DDX of cushings

A

Hypothyroidism, depression, PCOS, T2DM

How well did you know this?
1
Not at all
2
3
4
5
Perfectly
21
Q

History and symptoms of Cushings

A
  • S/S = Upper body obesity with thin limbs, skin problems (acne + plethora), high BP, muscle./boen weakness, moodiness, high blood usgars, tachycardia.
  • History = PMG (obesity, diabetes), medications (long term steroids/frequent steroid courses)
How well did you know this?
1
Not at all
2
3
4
5
Perfectly
22
Q

RFs of Cushings syndrome

A

•RFs = Adrenal or pituitary tumours, long term therapy with corticosteroids and being female.

How well did you know this?
1
Not at all
2
3
4
5
Perfectly
23
Q

Management of Cushings syndrome

A
  • Iatrogenic – stop meds
  • Cushings disease – remove pituitary adenoma, bilateral adrenalectomy, possible radiotherapy
  • Adrenal adenoma/ carcinoma – adrenalectomy
  • Ectopic ACTH –s urgery to remove tumour if no mets
  • Medis – Metyrapone, ketonconozole, fluconazole
How well did you know this?
1
Not at all
2
3
4
5
Perfectly
24
Q

Invetsigations of Cushings sundrome

A

•= 24hour urinary cortisol. Late night salivary cortisol (elevated due to loss of diurnal variation), overnight dexamethasone suppression test (AM cortisol). Helps identify excess hypercortisolaemia but does not identify the cause. Scans are CT chest (ectopic ACTH) or abdo, MRI adrenals, MRI pituitary

How well did you know this?
1
Not at all
2
3
4
5
Perfectly
25
Q

What is pseudo cushings

A

•– clinical features + biochemical evidence of cortisol excess but not caused by hypothalamic pituitary axis (depression, alcohol excess, obesity).

How well did you know this?
1
Not at all
2
3
4
5
Perfectly
26
Q

What are the complications of cushings syndrome

A

•Hypertension, Diabetes, Osteoporosis (reduces bone formation), prone to infections (suppressed immune system) . Cortisol does have mineralocorticoid activity so can cause electrolyte abnormalities (ihgh Na+, low K+, high BP).

How well did you know this?
1
Not at all
2
3
4
5
Perfectly
27
Q

Thyroid Function Tests - what are the categories and how to interpret

A
  • Look at Free T3/T4.
  • In any systemic illness, TFTs can be deranged and all low.
  • Thyroid autoantibodies – Antithyroid peroxidase (TPO) or antithyroglobulin Abs nay be increased in autoimmune thyroid disease, Hasimoto’s or Graves.
  • TSH receptor antibody – Mya be increased in Graves
  • Serum thyroglobulin – used for monitoring treatment carcinoma and detecting facticioushyperthyroidism
  • US – distinguishes cystic from soli (Poss malignant) nodules.
  • Isotope scan – for cause and detect retrosternal goitre thyroid metastases etc.
How well did you know this?
1
Not at all
2
3
4
5
Perfectly
28
Q

How is the Thyroid controlled

A
How well did you know this?
1
Not at all
2
3
4
5
Perfectly
29
Q

What is subclinical hypo and hyperthyroidism

A

Subclinical hypothyroidism: TSH high, Normal T4/T3 and no symptoms. Risk progression.

Subclinical hyperthyroidism: Decrease TSH, normal T4/T3.

How well did you know this?
1
Not at all
2
3
4
5
Perfectly
30
Q

What imaging to do you for the thyroid

A

Imaging in Thyroid: US (good for size + structure, FNA, scoring), CT scan and isotope scanning (structure + function)

How well did you know this?
1
Not at all
2
3
4
5
Perfectly
31
Q

Thyroid storm - describe this thyroid emergency - RFs, clinical features,

A
  • Thyroid storm: rare, potentially life threatening. 30-50% mortality.
  • RFs = acute infection, post partum, withdrawal of ATDs, recent surgery or RAI, radiographic contrast agents
  • Clinical features = altered mental state, severe hyperthyroid signs, vomiting, diarrhea, jaundice, pyrexia, tachycardia/tachyarrhythmia
  • Multisystem decompensation = cardiac failure, congestive hepatomegaly, resp distress, dehydration+ pre-renal failure
  • Tx = Betablockers, steroids, ATDs
How well did you know this?
1
Not at all
2
3
4
5
Perfectly
32
Q

Myxoedema coma - thyroid emergency

A

•Myxoedema Coma: Extreme hypo

  • Profound Hypothyroidism, Hypothermia, hyporeflexia, hypoglycaemia, bradycardia, seizures
  • Cardiomegaly + pericardial effusion, cerebella ataxia
  • Psychiatric symptoms (depression/psychosis), hyponatreamia, treated with IV Liothyronine (T3)- caution IHD.
How well did you know this?
1
Not at all
2
3
4
5
Perfectly
33
Q

Describe Thyroid hormone synthesis

A
  • Thyroid follicles is location
  • Iodine converted to I- by enzyme TPO
  • I- then attaches to tyrosine units of thyroglobulin
  • Proteases cleave T3 + T4 under regulation of TSH
  • In peripheral tissue de iodinases allow for conversion of T4->T3 (mor eptoent)
  • T3 hormone is able to affect transcription
  • T3 has greatest effect on neg feedback
  • TRH -> TSH -> T4 + T3
How well did you know this?
1
Not at all
2
3
4
5
Perfectly
34
Q

Hyperthyroidism - what is it and the sysmptoms and signs

A

•Hyperthyroidism is increased thyroid hormone synthesis and secretion specifically from disorders of thyroid gland so excess T3/54 and compensatory decrease in TSH. Unless ademona then increase TSH and increase T3/T4.

•Symptoms: Diarrhea, decreased weight, increased appetite, over active, sweats, heat intolerance, palpitations, tremor, irritability, labile emotions, oligomenorrhoea +/-infertility. Rarely psychosis, chorea, panic, itch, alopecia, urticaria.

•Signs: Pulse false/irregular, warm moist skin, fine tremor, palmar erythema, thin hair, lid lag, lid retraction. May be goitre, thyroid nodules.

How well did you know this?
1
Not at all
2
3
4
5
Perfectly
35
Q

Tests for Hyperthyroidism

A

•Tests: Decreased TSH,/T4 and increased T3. May be mild neutropenia (Graves)< Increased ESR/Ca+/LFTs. Check thyroid autoantibodies, isoptopescan and test eyes.

How well did you know this?
1
Not at all
2
3
4
5
Perfectly
36
Q

Causes of Hyperthyroidism

A

•Causes: Graves disease, Toxic multinodular goitre, Toxic adenoma, Ectopic thyroidtissuem Ecogenous.

How well did you know this?
1
Not at all
2
3
4
5
Perfectly
37
Q

Treatment fo hyperthyroidism

A

1.Drugs: B blockers (eg propranolol) for rapid control symptoms. 2 strategies of Anti thyroid meds. In Graves maintain either 12-18 tgen withdraw. Watch for agranulocytosis with carbimazole. Titration eg, carbimazole PO for 4wks then reduce according to TFT every 1-2m .Block replace: Carbimazole + levothyroxine together.

2.Radioiodine: Most become hypothyroid after. No ev for increased cancer, birth defects or infertility. Caution inactive hyper as risk of thyroid storm

3.Thyroidectomy: usually total but risk of damage to recurrent laryngeal nerve (hoarse voice) and hypoparathyroidism. Patients will beom hypothyroidism so thyroid replacement needed

4.In pregnancy + infancy: Get expert help.

How well did you know this?
1
Not at all
2
3
4
5
Perfectly
38
Q

Compications in Hyperthyroidism

A

Complications: Heart failure, angina, AF, osteoporosis, opthalmopathy, gynaecomastia.

Increase in BMR -> Hyperthyroidism -> weight loss

How well did you know this?
1
Not at all
2
3
4
5
Perfectly
39
Q

Causes of hyperthyroidism

A
  • Graves diseases
  • Toxic Multinodular goitre: In elderly and in iodine def areas. Nodules that secrete thyroid hormones. Sugrery indicated for compressice symptoms from enlarged thyroid
  • Toxic adenoma: Solitary nodule producing T3 &T4. On isoptope scan, the nodule is ‘hot’ and rets of gland suppressed.
  • Ectopic thyroid disease: metastatic follicular thyroid cancer, or struma ovarii: ovarian teratoma with thyroid tissue
  • Exogenous: Iodine excess eg, food contamination, contrast media. Levothyroxine excess cusesIncrease T4/D T3/ D thyroglobulin. Also Subacute de Quervain’s thyroiditis (self limiting post viral with painful goitre, Increase temp, +/- Increase ESR + low isotope uptake on scan), Drugs (amiodarone lithium), postpartum, TB.
How well did you know this?
1
Not at all
2
3
4
5
Perfectly
40
Q

WHat are the signs and triggers of graves disease

A
  • Signs: Eye disease (exophthalmos, opthalmoplegia), Pretibial myxoedema, Thyroid acropachy (clubbing, painful finger + toe swelling…
  • More males, typical 40-60. Cause is circulating IgG autoantibodies binding to and activating G coupled thyrotropin receptors, which cause smooth thyroid enlargement and increased hormone production and react with orbital autoantigens.
  • Triggers – stress infection, childbirth. Parents often hyperthyroid. Associated with other autoimmune diseases (Vitiligo, T1DM, Addison’s,
How well did you know this?
1
Not at all
2
3
4
5
Perfectly
41
Q

What are the causes of hypothyroidism?

A
  • Primary autoimmune hypothyroidism - primary atorphic and Hashimotos thyroiditis (goitre).
  • Primary hypoT - Iodine deficiency, post-thyroidectomy or radioiodine treatment, drug induced 9antithyroid drugs), subacute thyroiditis (temp hypoT after hyperthyroid phase).
  • Secondary hypoT - not enough TSH (hypopituitarism)
How well did you know this?
1
Not at all
2
3
4
5
Perfectly
42
Q

Hyponatraemia - def, causes, clinical features

A
  • Serum Na+ <135
  • XS water compared to electrolyts so low plasma osmolarity
  • Causes: Increeased reabsorption (Cirrhosis, CHF, Nephrotic syndrome), Reduced excretion (SIADH), Salt def (renal/non renal loss)
  • clinical = Water excess - from brain injury(confusion, headache, seizures), Hypervolaemia confined to ECG so oedema + fluid overload, salt def present with hypovolaemia + tachycardia with postural hypotension.
How well did you know this?
1
Not at all
2
3
4
5
Perfectly
43
Q

Investigations and treatment for hyponatraemia

A

Ix = rine Na+ (response of kidneys to low Na+ in blood), TFT (hypothyroidism), Cortisol (Addison’s), Glucose (hyperglycaemia-DKA often), Urine and serum osmolarity (mainly determined by Na+), serum one automatically low), LFTs.

Tx:

  • salt deficient – iv saline
  • XS fluid – fluid restriction (500-750ml/24h)
How well did you know this?
1
Not at all
2
3
4
5
Perfectly
44
Q

WHta are the causes, def and treatment of SIADH

A
  • Causes: Tumors, chest disease, metabolic, CNS disorders,, drugs, idiopathic
  • Dx criteria: Decrease Na+ serum osmolarity <270, inappropriate urine osmolarity, excess renal sodium loss >30, absence clinical evidence of hypovolaemia or fluid overload, normal renal/adrenal/thyroid function.
  • Tx: Treat underlying cause, fluid restrict (500-700ml/24hr), drug Tx (demeclocycline-induces partial nephrogenic DI, and vasopressin V2 receptor antagonist Tolvaptan promotes aquapheresis but expensive). Saline infusion in emergency (vom/sizures/GCS<8/CR distress, 150ml 3%saline over 20mins, aim for initial rise 5mmol/L.
How well did you know this?
1
Not at all
2
3
4
5
Perfectly
45
Q

What is Cerebral salt wasting

A
  • Potential cause of hyponatremia in CNS disease. Hyponatremia and ECF depletion from inappropriate sodium wasting in urine.
  • CSWS- low blood sodium conc and dehydration in response to injury or tumor’s surrounding brain.
How well did you know this?
1
Not at all
2
3
4
5
Perfectly
46
Q

Hypernatramia - cause, def, complications and Tx

A

Serum Na+>145mmol/L

  • Usually from loss of water in excess of loss of Na+. Caused by vomiting, diarrhea, diuretics,diabetes,iV hypertonic saline, IV NaHCO3, DI, Cushing’S.
  • Complications: seizures, subdural+intracranial haemorrhage, ischaemic stroke, dural sinus thrombosis.
  • Use 0..9% IV saline to replace volume – 1L over 8-12hrs. Swithc to 0.45% saline or 5%glucose once euvolaemic
  • Ix = U&Es, glucose, urea and creatinine, urine + serum osmolarity
How well did you know this?
1
Not at all
2
3
4
5
Perfectly
47
Q

What is Hyperkalamia and the causes

A

K+ >5.5mmol/L

Causes:

  • Decrease renal excretion – AKI/CKD/ K Sparing diuretics (spironolactone)
  • Cell injury – rhabdomyolysis/burns/blood transfusion/tumor cell necrosis
  • K+ cellular shifts – acidosis/drugs (suxamethionium)/ Tumour cell necrosis
  • Hyperaldosteronism – Addison’s/ drug induced (NSAIDs/ACEi)
  • Spurious
How well did you know this?
1
Not at all
2
3
4
5
Perfectly
48
Q

What are the symptpms and ECG changes with Hyperkalaemia then how to do manage these

A
  • Clinical = muscle weakness and cramps; paraethesia, hypotonia, focal neuro deficits. Oft asymptomatic.
  • Mx =Review ECG, 10% calcium chloride to stabilize myocardium, IV glucose/insulin infusion to bring K+ into cells. Treated fluid deficiency and underlying cause eg, Addison’s disease
How well did you know this?
1
Not at all
2
3
4
5
Perfectly
49
Q

Hypokalaemia - def, causes

A
  • K<3.5
  • •Causes – alcohol excess/ CKD/ Diuretics/ DKA/ Diarrhea/ LaxatiVes/ vomiting/ primary hyperaldosteronism/ Cushing’s syndrome
How well did you know this?
1
Not at all
2
3
4
5
Perfectly
50
Q

Symptoms, ECG and treatment of hypokalaemia

A
  • Symptoms = May cause lethargy, weakness, leg cramps
  • ECG- flatted T waves and prominent U waves, QT prolongation
  • Treat = replace. PO potassium chloride if mild. Iv fluids with KCI if severe
  • Correct Mg deficiency if present in patients with severe hypokalaemia.
How well did you know this?
1
Not at all
2
3
4
5
Perfectly
51
Q

Hypercalcaemia -

A

•Symptoms = Renal (polyuria, polydipsia), GI (anorexia, vom, consitpaiton, abdo pain, gastric ulcers, pancreatitis), CNS (confusion, lethargy, depression), other (pruritis, sore eyes). Also consider bone pain, fracture, hematuria, loin pain (renal stones).

•Causes = hyperparathyroid (1/3ry), malignancy (humoral hypercalcaemi, multiple myeloma, bony mets), vit D toxicity, BFHH, Sarcoidosis + other granulomatous disease, drugs (thiazide diuretics, lithium), immobialisation, hyperthyroidism, renal failure, Addison’s, vit A toxicity.

•Mx = Check vit D, PTH, TFTs. IVT 0.9% saline, steroids (prednisolone, dexamethasone and IV Zolendronic acid.

How well did you know this?
1
Not at all
2
3
4
5
Perfectly
52
Q

Hypocalcaemia - causes

A
  • Ca2+ homeostasis controlled by PTH + Vitamin D levels.
  • Causes: Hypoparathyroidism, vitamin D deficiency,, hypomagnesium
  • Ca2+ homeostasis: Calcitriol – Released when blood plasma (Ca2+) = low, increases Ca2+ bone resorption, absorption of Ca2+ from small intestines.
How well did you know this?
1
Not at all
2
3
4
5
Perfectly
53
Q

What are the 2 main lineages in haematopoiesis

A

Lymphoid or myeloid

How well did you know this?
1
Not at all
2
3
4
5
Perfectly
54
Q

What is anaemia and what symptoms and signs might your expect

A

Anaemia = Low haemaglobin conc which may be due to low red cell mass or increased plasma volume. Low Hb is <135men, <115 women.

  • Symptoms – due to underlying cause or anaemia itself: fatigue, dyspnoea, faintness, palpitations, headache, tinnitus, anorexia.
  • Signs – may be pallor, signs hyperdynamic circulation (eg tachy), flow murmurs, cardiac enlargement nad later HF.
How well did you know this?
1
Not at all
2
3
4
5
Perfectly
55
Q

What types of anaemias are associated with Low/Normal/High MCV

A
  • Low MCV = Iron def, thalassaemia, sideroblastic
  • Normal MCV = acute blood loss, anaemic chronic disease, bone marrow failure, renal failure, hypothyroidism, haemolysis, pregnancy
  • High MCV = B12 or folate def, (can alter DNA synthesis) alcohol excess (or liver disease), reticulocytosis, cytotoxics, myelodysplastic syndromes, marrow infiltration, hypothyroidism. Antifolate drugs. Pernicious anaemia(autoimmune)
  • Haemolytic anaemias = don’t fit in above classification . Suspect if reticulocytosis, mild macocytosism decrease haptoglobin, increase bilirubin/LDH/urobilinogen. Often mildly haundiced.
  • Applastic anaemia - Caused by reduction in pluripotent stem cells so reduce all cell types.
How well did you know this?
1
Not at all
2
3
4
5
Perfectly
56
Q

What causes iron deficiency anaemia and how mgiht you present

A
  • Causes = Excessive blood loss (menorrhagia, colon cancer, post menopausal), inadequate dairy intke, poor intestinal absorption (coeliac), increased requirements
  • Signs = Chronic IDA (koilonychia, atrophic glossitis, angular cheilosis and rarely post cricoid webbs).
  • Symptoms = Fatigue, SOB, Palpitations, pallor, nail changes, hair loss, atrophic glossitis, angular stomatitis.
How well did you know this?
1
Not at all
2
3
4
5
Perfectly
57
Q

What are the investigations for iron def anaemia

A
  • Blood film: microcytic, hypochromic anaemia with anisocytosis + poikilocytosis.
  • Decrease MCV/MCH/MCHC. High tIBC (relflects low iron stores)
  • Confirmed by decreased ferritin (acute phase protein which also increases with inflamm/malignancy.)
  • Check coeliac serology – if neg and not menstruating need urgent olonscopy and gastroscopy.
  • Endoscopy to rule out malignancy in unexplained.
How well did you know this?
1
Not at all
2
3
4
5
Perfectly
58
Q

Treatment for iron deficiency anaemia

A
  • Orla iron e,g ferrous sulphate but SE nausea/abdodiscomfort/constipation/diarrhea/ black stools. Hb should raise 10 a week. Continue till at least 3m after normalizes to replenish stores.
  • IV iron only if oral ineffective or impossible.
  • Check compliance
How well did you know this?
1
Not at all
2
3
4
5
Perfectly
59
Q

WHta can anaemia of chronic disease arise from?

How to test and how to treat?

A
  • Arises from 3 problems: Poor use iron in erythropoiesis, Cytokine induced shortening od RBC survival and Decrease production to and response to erythropoietin.
  • Causes: Chronic infections, vasculitis, rheumatoid, malignancy,r neal failure…
  • Tests: ferritin normal or increased in mild normocytic or microcytic anaemia. Check blood film, B12, folate, TSH, and tests for haemolysis.
  • Treatment – Underlying disease and erythropoietin.
How well did you know this?
1
Not at all
2
3
4
5
Perfectly
60
Q

Sideroblastic anaemia - what is this and when do you consider it?

A

: Red cells fail to completely form haem, whose biosynthesis takes place partly in mitochondrion. So deposits of iron in mitochondria that form ring around nucleus.

  • Think about this when microcytic and not responding to iron. Characterised by ineffective erythropoiesis, leading to increased iron absorption, iron loading in marrow and possible haemosiderosis (endocrine, liver and heart damage from iron deposition).
  • Causes: Congenital (rare, x linked), or acquired eg, idiopathic eg myelodysplastic disease, follow chemo, alcohol…).
  • Tests: look for increase ferritin, hypochromic blood film and disease defining sideroblasts in marrow.
  • Treatment: remove cause. Pyridoxine and possible repeated transfusions for severe anaemia.
How well did you know this?
1
Not at all
2
3
4
5
Perfectly
61
Q

Different types nutrition

A
  • Enteral nutrition – If gut working normally then preferred way.
  • By mouth – nutritional products
  • Through tube (NG/NJ…)
  • Parenteral nutrition – if gut cant absorb nutrients then deliver to bloodstream through drip.
  • PEG tube – feeding tube through skin and stomach wall.
How well did you know this?
1
Not at all
2
3
4
5
Perfectly
62
Q

Describe te glomerulus and its function

A

Glomerulus = loop of capillaries surrounded by bowman capsule.

  • Filtration barrier has 3 layers: Endothelial cells, basement membrane + podocytes of bowman capsule
  • Ultrafiltration = 1st stage in urine production occur here.
  • Afferent arteriole dilation + efferent arteriole constriction -> pressure gradient allowing filtration across barrier.
  • Nephortic syndrome = proteinuria, hypoalbuminaemia + oedema. Minimal change disease (podocyte pathology) is most common cause.
  • IgA nephropathy – IgA mediated inflammation
How well did you know this?
1
Not at all
2
3
4
5
Perfectly
63
Q

Descrieb the PCT and the function

A
  • High capacity for reabsorption, has epithelial cells, large channels for ion transport.
  • Pars convolute + pars recta
  • Reabsorption via paracellular + transcellular transport
  • Na+ drives reabsorption of other substances eg, water, glucose
  • Secretion_ substances removed from blood in PCT.
How well did you know this?
1
Not at all
2
3
4
5
Perfectly
64
Q

Descrieb the Part sof the Loop of henle and their function

A

Absroption of Na+

  • Thin DL - highly permeable to water – water reabsorption occurs driven by counter currently multiplier system
  • Thin AL – Impermeable to water, reabsortopn of Na+ and Cl-
  • Thick AL – primary site of Na+ reabsorption and impermeable to water – NKCC2 transporter
How well did you know this?
1
Not at all
2
3
4
5
Perfectly
65
Q

bartter syndrome

A

Bartter syndrome: autosomal recessive disease -> mutations in NKCC2, K+/Cl- channels -> hyponatraemia, hypokalaemia + metabolic alkalosis.

How well did you know this?
1
Not at all
2
3
4
5
Perfectly
66
Q

Early DCT - cells here + urpose

A

Early DCT: Impermeable to water, reabsorption of Na+,Cl-, Ca2+. NC channel transporter

•Macular densa cells as part of tubuloglomerular feedback to regulate GFR + blood flow

How well did you know this?
1
Not at all
2
3
4
5
Perfectly
67
Q

Late DCT and cells here

A

Late DCT: mostly principal cells + intercalated cells.

  • Principal cells – Na+ uptake + K_ excretion driven by ATPases + EHaca
  • Intercalated cells – control H+ and HCO3- conc.
How well did you know this?
1
Not at all
2
3
4
5
Perfectly
68
Q

Colelcting fuct function

A

Collecting duct: ADH + aquaporins to assist in reabsorption of water

•ADH: binds to V2 receptors -> adenylate cyclase -> cAMP -> vesicles containing aquaporin channels insert into apical membrane -> increased water reabsorption.

How well did you know this?
1
Not at all
2
3
4
5
Perfectly
69
Q

Diabetes insiidus

A

Polyuria + polydipsia. Insufficient ADH/ lack of response of collecting ducts to ADH. Less water reabsorbed formf iltrate -> increased volume of filtrate ->increased urine volume. Water deprivation test to confirm diagnosis.

How well did you know this?
1
Not at all
2
3
4
5
Perfectly
70
Q

SIADH effect of kidney nephron

A

SIADH: XS ADH released -> increased aquaporin expression in CD -> water retention. Dilution of blood -> Lowers Na+ -> hyponatraemia. Can be caused by ectopic ADH (paraneoplastic) from SCLC.

How well did you know this?
1
Not at all
2
3
4
5
Perfectly
71
Q

Npehortic syndrome - what is it and why does it happen?

A

Def = Triad of proteinuria >3g/24h, Hypoalbuminaemia usually <30g/L, oedema.

•Aetiology = Primary renal disease (membranous nephropathy(adults), focal segmental glomerulosclerosis, minimal change disease(kids)) . Secondary to systemic disorder (DM, lupus nephritis, myeloma, amyloid, pre-eclampsia, infections, genetic conditions, some allergic reactions. In young person thin about autoimmune renal disease.

•Patho = Filtration barrier of the kidney is formed by podocytes, the glomerular basement membrane and endothelial cells. Portein uria results form podocyte patho, abnormal function in minimal change disease, immune mediated damage in membranous nephropahy and podocyte injury/death in FSGS or patho in GBM/endothelial cell, membranoproliferative GN.

How well did you know this?
1
Not at all
2
3
4
5
Perfectly
72
Q

Presentation and DDx of nephrotic syndrome

A
  • Presentation = Generalized, pitting oedema, which can be rapid and severe. Look in dependent areas and those of low tissue resistance. Ask about systemic symptoms eg joint, skin. Consider malignancy and chronic infection.
  • DDx– minimal change nephropathy, FSGS, membranous nephropathy, diabetic nephropathy, primary glomerula disease,s, fibrillary glomerulopathues, lupus neprhtitis and multiple myeloma.
How well did you know this?
1
Not at all
2
3
4
5
Perfectly
73
Q

Ix of Npehortic syndrp,e

A

•24hour urine collection+ urine dip . Obs, DO bloods (FBC, U&Es– high urea poss, high creatinine,high K+, poss kidney US for DDx, poss renal biopsy. Check triglycerides and cholesterol as these are complications.

How well did you know this?
1
Not at all
2
3
4
5
Perfectly
74
Q

Mx of nephrotic syndrome and complications

A

: if secondary think about lifestyle control, DM etc. refer to nephrologist.

  • Reduce oedema: Fluid and salt restriction. Diuresis with lop diuretics eg, furosemide. Aim 0.5-1kg weight loss per day to avoid IV vol depletion + secondary AKI Thiazide diuretics can be added.
  • Treat underlying disease: Renal biopsy in adults and in children steroids induce remission in most unless no response to steroids or clinical feature suggesting another cause then biopsy. Uuslaly biopsy as last effort if unclear why
  • Reduce Protein uria: ACEi/ARB reduce proteinuria (may not be needed)
  • Complications: Thromboembolism, infection, hyperlipidaemia, acute renal failure as hypercoagubale. Can also get malnourished if dotn eat due to it.
How well did you know this?
1
Not at all
2
3
4
5
Perfectly
75
Q

Diabetic npehorpathy - what is it, symptoms, tests and how to treat

A
  • Diabetic nephropathy.- kidney damage caused by diabetes (kidney disease).high blood glucose damages the small blood vessels and tiny filters in your kidneys so they leak and don’t work as well. Abnormal amounts protein from blood can leave your body in your urine.
  • Symptoms = Swollen ankles/feet/hands, blood in pee, fatigue, SOB, feeling sick.
  • Tests = Urine test (ACR), blood test (eGFR)
  • Treat = BP under control 9ACEi/ARBs),might need avoud some foods, latest option is kidney transplant.
How well did you know this?
1
Not at all
2
3
4
5
Perfectly
76
Q

What is chronci kidney disease, causes and classification

A
  • Def = Abnormal kidney structure of function, present or >3m with implications for health.
  • Classification = Based on GFR category, presence albuminuria as marker or kidney damage, and cause of kidney disease. The lower the GFR and albuminuria associated with higher mortality, CV mortality, kidney failure, AKI.
  • Most common causes = Diabetes, glomerulonephritis, Increased BP/renovascular disease
How well did you know this?
1
Not at all
2
3
4
5
Perfectly
77
Q

CKD invetsigations

A
  • Bloods – U&Es, Hb, (normochromic, normochytic anaemia), glucose (DM), Decrease Ca and increase PO4^3-/PTH). Directed iinvetsiagtions Urine – Dipstick, MC&S, A:CR, P:CR, Bence jones
  • Imaging – USS and exclude obstruction.

Histology: Consider renal biopsy in progressive disease, nphrotic syndrome, systemic disease, AKI without recovery. Unlikely to change treatment if GFR stable and P:CR

How well did you know this?
1
Not at all
2
3
4
5
Perfectly
78
Q

Treatment of Chronic kidney disease

A
  1. Appropriate referral to nephrology:
  2. Treatment to slow renal disease progression: Target BP (<140, <90), RAAS (ACE-i/ARB for DM/HBP/CD with high A:CR), Gkycaemic control, Lifestyle
  3. Treatment of renal complications CKD: Anaemia, acidosis (conside sodium bicarb supplements), oedema (restrict fluid + sodium intake- may need loop diuretics), CKD bone mineral disorders (as increase serum phosphate, reduced hydroxylation vit D by kidney), restless legs/cramps, Diet.
  4. Treatment other complications CKD: CV disease (antiplatelets, atorvastatin…)
  5. Prep for renal replacement therapy (dialysis/transplantation):All suitable ones should be listed for deceased donor transplantation 6m before anticipated start RRT.
How well did you know this?
1
Not at all
2
3
4
5
Perfectly
79
Q

Glomerulnephritis - what is it and how to IX and MX

A

•Glomerulonephritis = term that encompasses conditions which are caused by pathology in glomerulus; present with proteinuria haematuria, or both; diagnosed on renal biopsy, cause CKD, can progress to kidney failure

•Investigations: Assess damage + potential cause

•Bloods – FBC. CRP, U&E, LFT, immunoglobulins, electrophoresis complement, autoantibodies, blood culture, ASO, hepatitis serology

•Urine – MC&S, Bence Jones protein; A:CR/P:CR, RBC casts

•Imaging – CXR (pulm haemorrhages), renal US

•Renal biopsy: For diagnosis – Exam of glomerular lesions provides GN diagnsosi. Includes proportion glomeruli involves, how much, hypercellularity, sclerosis/ Electron microscopy for ultrastructure, immunhisotlog

•Mx – Gen management is BP control and inhibition of renin-angiotensin axis, Specific is immunosuppression etc.

How well did you know this?
1
Not at all
2
3
4
5
Perfectly
80
Q

Nephritic vs nephrtoic syndrome

A

•Nephrosis (proteinuria due to podocyte patho) or Nephritis (haematuria due to inflammatory damage). If GN causes scaring, can get proteinuria which can cause complications.

How well did you know this?
1
Not at all
2
3
4
5
Perfectly
81
Q

Nephritic glomerulonephrtitis- causes

A

•Proteinuria due to podocyte pathology

  • IgA nephropathy: Asymptomatic non visible hematuria or episodic visible haematuria which may be ‘; within 12-72h infection. Increase BP. Protein uria, slow indolent disease. Diagnosis by renal biopsy (IgA deposition in Mg). Treat with ACE-i/ARB reduce proteinuria + protect renal function.
  • Henoch-Schönlein Purpura (HSP): Small vessel vasculitis + systemic variant IgA nephrophathy with deposition of IgA in skin, joints alongside kidney. Purpuric rash on extensor surfaces, flitting polyarthritis, abdo pain, nephritis. Usually clinical Dx, confirmed with positive IF or igA and C3 in skin. Renal biopsy identical to igAnephropathy. Managed same as above but steroids for gut involvement.
  • Post-Streptococcal GN: After throat or skin infection where streptococcal antigen deposition in glomerulus leading to immune complex form and inflammation. Varies haematuria to acute nephritis, (increase BP/oliguria… EV streptococcal infection, need supportive antibiotics
  • Anti-glomerula basement membrane disease: rare. Autoantibodies to type iV collagen. Presents with renal disease and lung disease. Anti-GBM for Dx, nee dplamsa exchange, corticosteroids and cyclophosphamide.
  • Rpaidly protessive GN – Breas in GBM allow inflx inflammatory cells etc so cresecents seen on renal biopsy. Need corticosteorids and cyclophosphamide.
How well did you know this?
1
Not at all
2
3
4
5
Perfectly
82
Q

What are the vitamin K dependent clotting factors

A

2,7,9,10

How well did you know this?
1
Not at all
2
3
4
5
Perfectly
83
Q

What is DIC

A

•in severe systemic illness, dying cells release procoagulant agents that activate coagulation resulting in fibrin generation that occludes small vessels , platelets and clotting factors are used up and result in bleeding elsehere. Blood tests reveal thrombocytopenia, increased PT/INT and APTT, and raised D-dimer and fibrin degredation products. Treat by removing cause and supportive therapies (blood, platelets, FFP, cryoprecipitate).

How well did you know this?
1
Not at all
2
3
4
5
Perfectly
84
Q

Why might lvier coagualtion be impaired

A

•Coag impairment – Mechanistic: Chronic liver disease (cirrhosis/Malnutrition), acute liver failure, biliary patho (cholestatic liver disease).

How well did you know this?
1
Not at all
2
3
4
5
Perfectly
85
Q

Describe the structure of the liver and cells

A
  • Liver = 2 major lobes separated by falciform ligament and surrounded by Glisson’s capsule (fibrous CT). Within liver sinusoids are O2 poor and nutrient rich blood from portal vein and mixes with O2 blood from hepatic artery. From sinusoids, blood enters system of veins that converge forming hepatic veins. Enter IVC.
  • Endothelial cells separated from hepatocytes by sub endothelial space (Space of Disse/ Perisinusoidal sac).
  • Blood passes through endothelial wall to have intimate contact with hepatocyte surface facing perisinusoidal space
  • Proteins made by hepatocytes transferred to blood by perisinusoidal space
How well did you know this?
1
Not at all
2
3
4
5
Perfectly
86
Q

What is jaundice and the 3 maint ypes

A
  • Jaundice: Unconjugated BR is insoluble in water (only travels in blood if bound to albumin) and conjugated is soluble in water.
  • Prehepatic jaundice – Increased haemolytic -> increased unconjugated BR
  • Hepatic jaundice: Liver impairment. Decreased ability of liver to conjugate bilirubin so both UN and conjugated in blood
  • Post hepatic jaundice: Blockage of bile duct resulting in back flow of conjugated BR into blood.
How well did you know this?
1
Not at all
2
3
4
5
Perfectly
87
Q

Descrieb the production of bilirubin

A
How well did you know this?
1
Not at all
2
3
4
5
Perfectly
88
Q

What the main LFTs and intepretation

A
  • ALT - Hepatocyte.
  • AST - Hepatocyte
  • Prothrombin time - synthetic function
  • ALbumin - synthetic function
  • GGT, ALK P - outflow function (cholangiocytes)
How well did you know this?
1
Not at all
2
3
4
5
Perfectly
89
Q

Functions of the liver

A
  • Glucose metabolism
  • lipids - uptake, synthesis, pacakging
  • Portein and amino acids- anabolism
  • secretion of bile
  • Sotrage fo Vitsmin A,D,B12,Cu, Fe
  • Haematolgoical fucntion - phagocytosis, hemopoiesis
How well did you know this?
1
Not at all
2
3
4
5
Perfectly
90
Q

What is the primary epithelial cell in the liver

A

Hepatocytes

How well did you know this?
1
Not at all
2
3
4
5
Perfectly
91
Q

Kupfer cells

A

reisdent macrophages

How well did you know this?
1
Not at all
2
3
4
5
Perfectly
92
Q

SInusoidal cells

A

Specialised endothelial barrier functions

How well did you know this?
1
Not at all
2
3
4
5
Perfectly
93
Q

What are some coagualtion disorders that can cocur in liver disease

A
  • Unable to absorb Vitamin K : Obstructive biliary disease -> impaired clotting
  • Hepatocyte damage: Reduced clotting factors of intrinsic + extrinsic pathway.
  • Platelets: hypersplenism can reduce platelets
  • Biliary: Cholestatic obstructive disease: blockage of bile ducts -> reduced vitamin K absorption -> prolonged clotting time
  • Thrombocytopenia
  • DIC: Systemic illness where cells release procoagulant agents -> fibrin deposition -> vessel occlusion.
  • Bleeding in liver disease: Less absorption of vitamin K, vitamins ADEK (fat soluble) and need bile. Less sysnthesis of other lcotting dactors, low platelets, reduced levels protein c and S. Less fibrinogen, endothelial dysfunction
How well did you know this?
1
Not at all
2
3
4
5
Perfectly
94
Q

Classify liver disease into 2

A
  • Acute liver failure: Fulminant liver failure, uncommon but life threatening. Most commonly due to paracetamol toxicity in UK and acute viral hepatitis world wide. Presents with jaundice, encephalopathy and coagulopathy. Ascites uncommon, might need liver transplant. Coagulopathy/Hepatic encephalopathy/acute liver injury
  • Chronic liver Disease: Progressive destruction and regeneration of liver parenchyma and regeneration of liver parenchyma leading to cirrhosis and fibrosis. Cirrhosis is final common pathway for wide variety of insults to the liver. Often asymptomatic until late stage. Mostly from alcohol liver disease, chronic viral hepatitis or non alcoholic fatty liver disease.
How well did you know this?
1
Not at all
2
3
4
5
Perfectly
95
Q

Gastr-oeosphageal varices - what are these and the RF, sympotms

A
  • Abnormal, enlarged veins in the tube the osophagus. Most have chronic liver disease.
  • RF = Increase portal pressure, variceal size, endoscopic features od variceal wall and advanced liver disease.
  • Symptoms – haematemesis, melaena, abdo pain, dysphagia/odynophgia, confusion
  • Signs – Peripherally shut down, pallor, hypotension + tachycardia), reduced urine output, melaena, signs chronic liver disease, reduced GCS, signs sepsis.
How well did you know this?
1
Not at all
2
3
4
5
Perfectly
96
Q

Invetsiagtions and management of Oesophageal varices

A
  • Ix – FBC, INR, renal function, LFTs, BUN, group and cross match, hepatic serology, CXR, ascitic tap
  • Imaging – CT/MRI-a, IS, doppler sonography, video capsule endoscopy, hepatic vein pressure gradient, transient elastography
  • Mx – Band ligation for UGIB from varices, TIPS considered if not controlled by bands, stent insertion when other failed
How well did you know this?
1
Not at all
2
3
4
5
Perfectly
97
Q

Causes of portal hypertension

A
  • Preheptaic – thrombosis (portal ro splenic)
  • Intrahepatic – cirrhosis, schistosomiasis, sarcoid, myeloproliferative diseases, congenital hepatic fibrosis
  • Post hepatic – Budd-Chiari syndrome, RHF, constrictive pericarditis
How well did you know this?
1
Not at all
2
3
4
5
Perfectly
98
Q

Alcoholic liver disease - the stages

A
  • Fatty liver – Build up of fat inside liver cells. Leads to an enlarged liver. It’s the most common alcohol-induced liver problem. Often no symptoms, but might get tired/weak/weight loss.
  • Alcoholic hepatitis – Acute inflammation of the liver. Death of liver cells, often followed by permanent scarring. Pian over liver, fever, weakness, nausea & vomiting, appetite loss, jaundice.
  • Alcoholic cirrhosis – Destruction of normal liver tissue. It leaves scar tissue in place of working liver tissue. Portal hypertension, enlarged spleen, poor nutrition, bleeding in intestines, ascites, kidney failure, confusion, liver cancer.
  • Dx – Bloods (LFTs), Liver biopsy, US, CT scan, MRI
  • Rfs – high alcohol intake, binge drinking, female gender, genetic
How well did you know this?
1
Not at all
2
3
4
5
Perfectly
99
Q

What is alcohol dependency and signs of this

A
  • Alcohol dependency = Chronic medical condition that typically includes a current or past history of excessive drinking, strong craving for alcohol, continued use despite repeated problems with drinking and an ability to control alcohol consumption.
  • Patient – malaise, increase TPR, anorexia, D&V, tender hepatomegaly, possible jaundice, bleeding, ascites
  • Blood – Increase WCC, decrease platelets, Increase INR, Increase AST, Increase MCV, increase urea
  • Jaundice, encephalopathy or coagulopathy is severe hepatitis.
How well did you know this?
1
Not at all
2
3
4
5
Perfectly
100
Q

How to assess risk of alcohol dependency

A
  • Assessing risk – AUDIT-C: How often do you have a drink containing alcohol, how many units of alcohol do you drink on a typical day when you are drinking and how often if more than 6(female), more than 8 (male) on single occasional in last year.
  • 0-4 lower risk, 5-12 watch out. Possible referral if 11/12.
  • Advised not more than 14 units a week and spread over 3days or more if you are doing so.
How well did you know this?
1
Not at all
2
3
4
5
Perfectly
101
Q

Alcohol withdrawal syndrome - pathology and symptoms

A
  • AWS = sudden cessation or reduction in usual intake after long term use of alcohol, varies in severity and may represent medical emergency. Need effective Mx early. Not directly related to intake.
  • Imbalance in NT in brain caused by chronic consumption (GABA-inhibitory), NDMA (excitatory).
  • Alcohol acts on GABA receptors so I hibitory effects so to maintain homeostasis with downregulate GABA neurons + upregulate NDMA.
  • On cessation then imbalance NTs so hyper-excitability so AWS. Only drop alcohol levels required so still can have symptoms despite detectable blood alcohol level.
  • Minor symptom – Anxiety, insomnia -> Hallucinations -> seizures (as early as 2hours after) -> delirium tremens (agitation, tachyc, fever, hypertension – peak 5days-7days).
  • Clinically evaluate risk of thiamine deficiency = PIC
How well did you know this?
1
Not at all
2
3
4
5
Perfectly
102
Q

Wernickes encephalopathy

A
  • Wernicke’s Encephalopathy = consequence thiamine Vit B1 def (essential coenzyme in biochem pathways). As this develops -> enzymes, system dependent on thiamine function less well and cell death. Can lead to permanent brain damage if not treated.
  • Give IV pabrinex before dextrose (glucose) = so glucose more likely to utilize pabrinex to form ATP or else only increase lactic acid produced.
  • CIWA = Clinical institute withdrawal assessment – 10- item scale to measure severity alcohol withdrawal syndromes.
How well did you know this?
1
Not at all
2
3
4
5
Perfectly
103
Q

Management of alcohol withdrawal syndrome?

A
  • Mx:Most hospitalize, catheter, CVP monitor, screen infections, stop alcohol consumption (can use chlordiazepoxide for withdrawal stuff or lorazepam, vitamins K, optimize nutrition, don’t use low protein diet if encephalopathy Dialy weight, bloods, steroids possible.
  • Relapse – Acamprosate at help intense anxiety, insomnia, craving. Disulfiram ca be used for dependance as acetaldehyde build up which is unpleasant with alcohol like flushing etc.
How well did you know this?
1
Not at all
2
3
4
5
Perfectly
104
Q

WHat is diabetes mellitus and the consequences

A

DM = lack of or reduced effectiveness of endogenous insulin. Hyperglyaemia. Causes serious microvascular (retinopathy, nephropathy, neuropathy) or macrovascular problems (Stroke, MI, Renovasular disease, limb ischaemia).

Other causes:

  • Pancreatitis , surgery to remove pancreas
  • Destruction – cystic fibrosis, hemochromatosis + pancreatic cancer
  • Endocrine – Cushin’s disease, pheochromocytoma, hyperthyroidism, pregnancy
How well did you know this?
1
Not at all
2
3
4
5
Perfectly
105
Q

Compare T1DM.T2DM - epidemiology genetics, cause, presentation, associations and treatment

A
How well did you know this?
1
Not at all
2
3
4
5
Perfectly
106
Q

What are soem oral hypoglycaemia agents for treating dm

A

Oral Hypoglycameic Agents:

  • Metformin – Biguanide, increase insulin sensitivity + help loose weight.
  • DPP4 inhibitors/ gliptins (eg sitagliptin), block action DPP-4, enzumes that destroys hormone incretin
  • Glitazone: Increase insulin sensitivity, increase LFT.
  • Sulfonyurea: increase insulin secretion eg, gliclazide
  • SGLTi: Selective sodium-glucose co-transporter-2 inhibitor. Blocks reabsorption of glucose in kidneys and promotes excretion of excess glucose in urine.
107
Q

Glucagon like peptide analogues and when used in DM

A

Glucagon-like peptide analogues (exenatide, liraglutie): incretin mimetics which augument insulin release by S/C injection/ BMI>35, specific problems with obesity or <35 and insulin ahs significant occupation implications etc.

108
Q

T1DM Treatment

A
  • Educate on ho to self adjust with exercise, calories, carb counting, phone support modify diet, dose titration to target.
  • Subcut insulin: Ultra fast acting (novorapid) at start of meal or just after to macth. Isophane insulin, Pre mixed insulins (short + long), long acting recombinant human insulin analogues (inulin glargine- T1/T2) at bedtime. Insulin determirsimilar for overweight T2DM.
  • Common regiments: Biphasic (2x premixed by pen), QDS (before meals ultrafast + bedtime one), Once daily (before bed).
  • Dose adjustment for normal eating
  • Subcut in intercurrent illness - don’t stop, aa usually need more so check glucose.
  • Insulin pumps (continuous subcut) – when attempts to reach HbA1c with multiple daily injections have dialing hypoglcyaemia or unable to achieve target.
109
Q

T2DM management

A
110
Q

Complications of Diabetes

A
  • Microvascular: Diabetic Retinopathy, Diabetic Nephropathy, Diabetic Neuropathy (sexual dysfunction, Gastroparesis, Autonomic)
  • Macrovascular: Cardiac events (ACS/MI), Cerebrovascular events, peripheral vascular disease
  • Diabetic Neuropathy: Decrease sensation in stocking distribution, absent ankle jerks, neuropathic deformity, claw toes, rocker bottom sole. These all lead to increased mechanical stress and repeated joint injury. Swelling, instability, deformity. Check for ischaemia, foot ulceration, bony defects, infections. Manage with regular chiropody. Possible surgery if abscess or deep infection etc.
  • Other: Mental health (ie depression), poor wound healing, recurrent infections, gum disease, cancer, diabetic foot
  • Emergencies
111
Q
A
112
Q

How does diabetes affect the kidneys?

A

Diabetes damages blood vessels. Damage to the blood vessels affects filtration by the kidneys.

  • Poorly controlled diabetes leads to increased glucose in the urine
  • More glucose reaches glomerular capillaries and hence bowman space
  • Glucose reabsorption in the PCT via cotransporters with Na+
  • Increased Nacl reabsorption, drags water with it as NaCl is osmotically active so water reabsorption will also increase.
  • Decreased low to Macula densa cells, tubuloglomerular feedback causes vasodilation of afferent arteriole and vasoconstriction of efferent arteriole to increase GFR + hence glomerular pressure
  • The increased GP means more matrix is deposited in and hypertrophy of mesangial cells occurs. These cells proliferate causing sclerosis, this eventually compresses glomerular capillaries and decreases GFR -> kidney failure

Dx: Microalbuminuria = urine dip negative for protein but ACR>3mg/mmol. Incaes renal disease + increased vascular risk. Control diabetes -> BP control (ACEi/ARB) and Na= restriction + statin (reduce CVS risk).

  • Harms the blood vessels inside your kidneys: High sugar levels cause narrowed and clogged vessels and kidneys damage without enough blood therefore albumin passes through filters and ends up in the urine Harms nerves in body: may not be able to feel when bladder ull which can damage kidneys.
  • Urinary tract: If urine stays in bladder for long time, may get UTI because of bacteria as they grow rapidly in urine with high sugar level. They can spread to the kidneys
  • Can lead to CKD.
113
Q

Diabetic nephropathy - pathology

A

Hyperglycaemia -> RAAS activation, oxidative stress -> increased glomerular pressure -> podocyte damage + endothelial dysfunction. Albuminaemia is the first clinical sign. Glomerulosclerosis -> nodule formation -> fibrosis + loss of kidney function. Existin hypertension accelerates condition.

114
Q

WHta is hypoglycaemia and the signs of it.

A
  • Hypoglycemia = Plasma glucose <3mmol/L. OR BM <4
  • Symptoms =
  • Autonomic – Sweating, anxiety, hunger, tremor, palpitations, dizziness.
  • Neuroglycopenic – confusion, drowsiness, visual trouble, seizures, coma. Rarely focal symptoms. Personality change, restless, incoherence.
115
Q

Management of hYpoglycaemia?

A

•Mx = ABCDE, if conscious (glucogel, lucozade/orange juice, long acitng carb) and if unconscious (iV glucose 100-200ml 10%), 1M 1mg glucagon, if regain consciousness then oral glucose switch.

116
Q

Diabetic ketoacidosis -

A
  • Hyperglycemia (BG>11 or known DM), Ketonaemia (ketones>3 or significant), Acidosis (bicarb <15 and or venus ph<7.3)
  • Can be initial presentation T1DM or poorly controlled
  • S/S = Nausea, vomiting, abdo pain, pear drop breath, thirst, sweating, confusion/coma, cramps, dehydration, reduced urine output.
  • Mx = ABCDE, senior support, rehydration,, insulin, insulin and treat underlying, diabetic team input when stable.
117
Q

HHS- hyperosmolar hyperglycaemic state:

A
  • Similar to DKA, in T2DM with slower onset
  • hypovolaemia, marked hyperglycaemia (>30mmol/l) without significant hyperketonaemia<3 or acidosis >7,3Phand high serum osmolarity >320mOsm/kg
  • Treat similar to dka BUT INSULIN AT HALF RATE (0.05MLS/KG/HR)
118
Q

How does diabetes effect the kidneys

A
119
Q

Hypertension - what is it and the stages

A

Hypertension = We choose to select a value above which risk is significantly increased and the benefit of treatment is clear cut. Don’t rly on single reading, confirm with 24hr ambulatory BP monitoring or week of home readings. Diagnostic threshold is lower than about 135/85

120
Q

Symptoms and tests with hypertension

A
  • S/S = Usually symptomatic except malignant (headache, visual problems). Headache not more common than normal pop. Check CVS and retinopathy. Check for renal disease, radiofemoral delay, weak fem pulses, renal bruise, palpable kidneys, cushing’s. Also check end organ damage (proteinuria, retinopathy LVH).
  • Tests = Confirm with ABPM or home bP. For overall risk then fasting glucose, cholesterol. Look for end organ damage on ECG/ECHO, urine analysis. Exclude secondary from U&E, Ca. Also special ones.
  • White coat HBP – elevated clinic pressure only
121
Q

Whta is malignant or accelerated phase hypertension

A

Malignant or accelerated phase hypertension - rapid rise leading to vascular damage. Usually severe >200/>130, and bilateral retinal haemorrhages an exudates with possible papilledema. May precipitate acute kidney injury

122
Q

Primary vs secodnayr hypertension

A
  • Primary hypertension (95%)– cause unknown in most
  • Secondary hypertension:
    • Renal disease – most common. 75% from intrinsic renal disease: glomerulonephritis, polyarteritis nodosa (PSN), systemic sclerosis, chronic pyelonephritis, or polycystic kidneys. 25% from renovascular disease, mostly atheromatous or rarely fibromuscular dysplasia.
    • Endocrine disease – Cushing’s and Conn’s syndrome, phaechromocytoma, acromegaly, hyperparathyroidism
123
Q

Treatment of hypertension

A
  • Goal - <140/90 (<130/80 Diabetes, 150/90 for 80+). Reduce slowly
  • Lifestyle Changes – smoking, low fat, reduce alcohol/salt
  • Drugs: ACEi for diabetes, renal problems etc.
  • Malignant HBP – oral therapy reduce over days. Bed rest. Atenolol or long acting Ca2+ blockers.
124
Q

Choice of hypertension drug

A
125
Q

Malabsorption - causes, symptoms, signs

A
  • Causes: Coeliac disease, chronic pancreatitis, Crohn’s Disease. Rarer are decreased bile, pancreatic insufficient, small bowel mucosa, bacterial overgrowth, infection, intestinal hurry.
  • Symptoms: Diarrhea, decreased weight, lethargy, steatorrhoea, bloating
  • Def signs: Anaemia (Decreased Fe, B12, folate), bleeing disorders (decrease VitK), oedema (decrease protein), metabolic bone disease (decrease Vit D), neurological feature eg neuropathy.
126
Q

Invesyiagting posisbel malasbroption

A
127
Q

4 main mechanisms of why weigh tloss leads to malnutrition

A
  • Impaired intake – disease, anorexia, psychological
  • Impaired digestion – underlying disease eg, cancer/IBD, coeliac disease, pancreatic abnormalities
  • Increased MR eg, cachexia in cancer of cystic fibrosis
  • XS nutrient loss eg, short bowel syndrome
128
Q

What 2 processes can lead to malabsroption if interfered with?

A

Malabsorption can occur if disease interferes with either digestion or absorption:

  • Digestion – disorders preventing mixing of food with digestive enxymes eg, pancreatic disease; Insufficient production of pancreatic enzymes, decreased ile 9liver disease), XS stomach acid eg, SE syndrome, Bacterial overgrowth
  • Absorption – Diseases injuring lining of intestine (infection/coeliacs), surgical removal of intestine (short bowel syndrome), disorders affect flow of lymph from bowel.
129
Q

MUST

A
130
Q

Vitamins and their deficiency syndrome

A
131
Q

minerals and their deficiency syndromes

A
132
Q

Appropriate invetsiagtions ofr malnutrition

A
  • MUST screening tool
  • U&Es (electrolytes+ risk refeeding syndrome)
  • FBC
  • Vitamin B12 + folate
  • Clotting screen – Vit K def
  • Stool tests – Faecal calprotectin, elastase
  • Sigmoidoscopy/colonoscopy
  • CRP/ESR
  • US, barium meal, CT/MRI
  • Hydrogen breath tests
133
Q

WHy do you get acromegaly and how do you test for it?

A
  • Patho = Due to increased secretion of GH from pituitary tumor or hyperplasia. GH stimulates bone and soft tissue growth through increased secretion of insulin like growth factor-1 (IGF-1)
  • Tests = increase glucose, increase Calcium, increase PO4-. Don’t rely on random GH d it in stress, sleep, puberty , pregnancy. Normally GH secretion inhibited by high glucose and hardy detectable but here it isn’t suppressed. May need OGTT if high GH/IGF-1. Collect glucose samples difference intervals (may get false pos with puberty preg etc). MRI scan pituitary fossa, look hypopituitarism, visual fields + acuity, ECG,ECHO.
134
Q

Treatment and ocmplications of acromegaly

A
  • Complications = (may present with CCF or ketoacidosis). Impaired glucose tolerance, vascular (increase BP, Lvhypertrophy, cardiomyopathy, arrhythmias, increased risk IHD + stroke. Neoplasia (increase colon cancer risk, may need colonoscopy). In preg- monitor glucose.
  • Tx = Aim to correct/prevent tumoru compression by excising lesion + reduce GH=IGF-1 <2.

1) transsphenoidal surgery,2) if fails then somatostatin analogues (SSA) and pos radiotherapy then 3) GH antagonist pegvisomat if resistant or intolerance to SSA. Follow up years levels etc, BMI.

135
Q

What are the signs and symptoms of acromegaly

A

Symptoms = Acroparaesthesia, amenorrhoea, decrease libido, headache, increased sweating, snoring, arthralgia, backache. Rinsg don’t fit etc.

Signs = Often predate diagnosis by 4years. Gigantism if it occurs before bony epiphyses fuse (rare).

136
Q

Inflammatory causes of GI disease

A

Inflammatory:UC/ CD

  • Symptoms = Peristsent diarrhea, abdo pain, rectal bleeding/bloody stools, weight loss,f tigue
  • Diagnosed = Endoscopy/ colonscopy, imaging studies, stool samples (fecal calprotectin)
  • Crohn’s = Transmrual granulomatous inflammation affecting GI tract. Skin lesions and cobblestone mucosa appearance
  • UC = Only affects colon, characterized by superficial inflammation and can affect part or all of colon (pancolitis). Punctate ulcers + hyperaemia/haemorrhagic colonic mucosa + pseudopolyps caused by inflammation
137
Q

Non inflammatory GI diseases

A

Colitis, IBS (increased WBC)

138
Q

IBS - what is it, diagnosis and treatment

A
  • IBS = Irritable bowel syndrome. Mixed abdo symptoms with no organic cause found, most likely due to disorders intestinal motility, enhance visceral perception or microbial dysbiosis.
  • Dx – recurrent abdo pain associated with at least 2: Relief by defecation, altered stool form, altered bowel frequency. Other features are urgency, incomple eva, bloating etc. Symptoimsfor >6m (chronic), may have general bdominal tenderness.
  • Check bloods, coeliac serology, colonscopy, refer for ovrian/bowel cancer, check stools. Might refer if diagnostic uncertainty, changing symptoms for known IBSm refractory to management or refractory symptom, chronic pain ovelrpa syndromes etc.
  • Tx – control symptoms like constipation, diarrhea, colic/bloating, psychological.
139
Q

Bowel cancer symptoms

A
140
Q

Complications of endoscopies

A

•Complications= Perforation, reaction to sedation, infection, bleeding, pancreatitis as result ERCP.

141
Q

WHta is the pathology of IBD

A
  • Ulcerative colitis, Crohn’s disease, Intermediate colitis, proctitis, microscopic colitis (collagenous colitis, lymphocytic colitis)
  • Patho: GIT is chronically exposed to various antigens found in bacteria and food. In a normal bowel, gut homeostasis is maintained by suppressing excessive immune responses to those foreign antigens. IBD is idiopathic disorder caused by chronic and excessive inflamm of GIT
142
Q

Diagnosis of IBD and extraintestinal manifestations

A
  • Dx – Symptoms (pain, weight loss, or bleeding, frequent, urgent loose motion, constipation perforation), FCP, Bloods (ESR/CRP.IDA), Endoscopy + biopsy, MRI, Abdo xray (toxic megacolon)
  • Extraintestinal manifestations: Autoimmune pancreatitis, autoimmune hepatitis, primary sclerosing cholangitis, erythema nodosum, pyodermal gangrenosum, anylosing psondylitits, episcleritis, pneumonitis, CV disease…
143
Q

DDx of iBD

A
  • Drug induced colitis – NSAIDs, PPI, Chemo, carbamazepine, diuretics, sertraline, cocaine..
  • Post infectious colitis – Cdiff, CMV, TB, Ecoli, campylobacter, shigella, parasites, STIs
  • Radiation proctitis – pelvic radiation, often <3m post
  • Ischaemic colitis – arterial thrombosis, HF, Emboli, small vessel disease, atherosclerosis, trauma, hypovolemia, vasculitis, drugs, radiation, volvulu
  • IBS- Normal FCP, Normal Endo, emotional, IBS+ Piles presents like iBD
144
Q

Treatments of IBD

A
145
Q

In severe CD what is the treatment

A
146
Q

Steroids in IBD treatment

A
147
Q

Why does iBD lead to weight loss

A
  • Can impact your ability to properly digest food and absorb nutrients, which may lead to serious vitamin deficiencies and malnutrition. (IBD causes damage to intestinal lining so interferes with absorption of nutrients -> malabsorption -> weight loss)
  • Severe diarrhea – depleted fluids, nutrients, electrolytes
  • Abdominal pain + nausea – reduce appeptite
  • Frequent bowel movements
148
Q

UC VS CD

A

(iBS might improve when not stressed but iBD never improves)

149
Q

WHat is UC and the appearance

A
  • Relapsing and remitting inflammatory disorder of the colonic mucosa. May affect the rectum or extend to colon.
  • Cause = inappropriate immune response against colonic flora in genetically susceptible individuals.
  • Patho = Hyperaemic/Haemorrhagic colonic mucosa +/- pseudopolyps formed by inflammation. Punctate ulcers may expend deep. Continuous inflammation limited to mucosa.
150
Q

SYmptoms and signs of UC

A
  • Symptoms = Fever, abdominal cramps, fatigue, bloody diarrhea or loose stool with pus, constant urge to pass stool. Loss of weight.
  • Sign = Tachycardia, tender/distended abdomen, fever, Extra-intestinal (clubbing, oral/ aphthous ulcers, erythema nodosum, conjunctivitis, iritis, episcleritis, pyoderma gangrenosum, large joint arthritis, sacroiliitis, ankylosing spondylitis, nutritional defects
151
Q

Tests for UC

A
  • Bloods: FBC, ESR, CRP, U&E, LFT, Blood culture
  • Stool: MC&S, CDT (exclude Campylobacter, C. difficile, Salmonella, Shigella, E.Coli, amoebae
  • Faecal Calprotectin for GI inflammation (but things like NSAIDs can give false positive)
  • AXR: No faecal shadows, mucosal thickening/islands. Colonic dilation
  • Lower GI endoscopy: limited flexible sigmoidoscopy if acute to assess and biopsy (full colonoscopy once controlled to define disease extent).
152
Q

Crohns disease de,f cause and associations

A
  • Def = Inflammatory disease characterized by transmural granulomatous inflammation affecting any part of the gut from mouth to anus.
  • Cause = Inappropriate immune response against the gut flora in a genetically susceptible individual.
  • Associations = Smoking, NSAIDs may exacerbate.
153
Q

SYmptoms and signs of Crohns disease

A
  • Symptoms = Diarrhea, loss of appetite, fatigue, stomach aches and cramps, weight loss, fever
  • Signs = Skin/joint, eye problems, anal strictures, abdominal tenderness/mass, bowel ulceration, clubbing, perianal abscess/ fistulae/ skin tages
154
Q

Tests and treatment for Crohns

A

Tests:

  • Bloods:
  • Colonoscopy + biopsy
  • Stool: MC&S, CDT, faecal calprotectin
  • Small Bowel: for detecting isolated proximal disease (eg capsule endoscopy). MRI (Pelvic disease, fistula, small bowel disease activity, strictures). US (for small bowel imaging if skilled operator

Treatments:

  • Optimise nutrition. Help them quit smoking .
  • Assess severity: Increase temp/ Pulse/ ESR/ WCC and decrease albumin. These may suggest IV steroids needed.
  • General iBD Treatment
155
Q

3 main symptoms to suspect Coeliac and the othe rpresentations

A
  • Suspect this if diarrhea + weight loss or anaemia
  • Presentation = (Can be asymptomatic). Atopy and presence of autoimmune disease.
  • Fatigue
  • GI : abdo pain, bloating , diarrhea, steatorhoea, weight loss, sub fertility, failure to thrive, nausea/vomiting
  • Atypical: Anaemia, angular stomitis, mouth ulcers, dermatitis herpetiformis
156
Q

What is coeliac disease and the genetic associations

A
  • T cell responses to gluten (alcohol soluble proteins in wheat, bareley, rye and possible oats) in small bowel causes villous atrophy and malabsorption.
  • Associations = HLA DQ2 (95%), rest DQ8, autoimmune disease, dermatitis herpetiformis
157
Q

Invetsigations and diagnosis of coeliac disease

A
  • Ix = FBC (anaemia, LFTs, Anti0endomysial and anti-tissue transglutaminase antibodies (TTG Ab), duodenal/ jejunal biopsy, DEXA scanning (in all as calcium and vit D malabsorption can lead to osteopenia).
  • Dx = Decrease Hb, Increase RDCWm decrease VB12, decrease ferritin. Antibodies – anti-transglutaminase is preferred (but its IgA antbodu). Where serology pos or high index suspicious then duodenal biopsy whole on gluten containing diet: villous atrophy, increased intra epithelial WBCs + crypt hyperplasia. Where doubt, HLA DQ2/8 genotyping might help.
158
Q

Treatment and complications of coeliacs

A
  • Tx = Lifelong gluten free diet. Monitor response by symptoms and repeat serology. Some oats may be tolerated. Dietician Mx, Iron,Ca2_, folate and vit B12 supplements.
  • Complications = Anaemia, dermatitis herpetiformis, osteopenia/osteoporosis, hyposplenism, GI T cells lymphoma, increased risk malignancy.
159
Q

EPidemiology for IBC,Pancreatic disease and coeliac disease

A
  • IBD: Crohns is white, age, 15-40, 60-80, FH of Crohns. UC is FH of IBD, HLA-B27, infection, NSAIDs, peaks at 15-30 and 0-70
  • Pancreatic disease: Eg pancreatitis typically 30-40/ Pancreatic cancer >45 with av age 70, pancreatic insufficiency >70, T1DM(first peak between 4-7 with second peak 10—14)
  • Coeliac disease – symptoms most likely to develop: during early childhood – between 8 and 12m old, although may take several years before correct diagnosis made. In later adulthood 40-60 years of age.
160
Q

Peptic ulcer Disease - symptoms, Tx, Complications

A
  • Symptoms = Epigastric pain often related to hunger, specific foods or time of day, fullness after meals, heartburn, tender epigastrium, GI bleeding
  • ALARM symptoms = Anaemia (iron def), Loss of weight, Anorexia, Recent onset/progressive symp, Melaena/hematemesis, Swallowing Difficulty
  • H.pylori – If <55 test and treat H.Pylori, if +ve then PP+2 Abs(lansoprazole, clarithromycin, amoxicillin). If –ve then just acid suppression. Refer urgent endo if dysphagia./
  • Tx: Lifetsyle (alcohol/tobacco), H.Pylori eradication (triple therapy), drugs to reduce acid (PPIS, H2 blockers), drug induced ucers (stop drug, PPIs and if symp persist re endoscopy).
  • Complications: Bleeding, perforation, malignancy, decrease gastric outflow.
  • Functional (non-ulcer) dyspepsia – common h.Pylori eradiation may help.
161
Q

Duodenal ulcer, RFs, symptoms

A

•Duodenal Ulcer: Major RFs are H.Pylori/drugs (NSAIDs, steroids/SSRI). Minor RFs re increased gastric acid secretion, increased gastric emptying (decreased duodenal pH), blood group 0, smoking.Symptoms are asymptomatic or epigastric pain (releievd by antacids), possible decrease weight, epigastric tenderness. Need Upper gI endoscopy, test for h.Pylori and measure gastrin conc when off PPIs If ZE suspected. Non ulcer dyspepsia is duodenal crohs, TB, lymphoma, pancreatic cancer. No follow up if good response to PPIs.

162
Q

Gastric ulcer- RFs, symptoms

A

•Gastric Ulcer: Mainly in elderly on lesser curve. RFs are H.Pylori, smoking, NSAIDs, reflux, delayed gastric emptying, stress. Aymtpomatic or epigastric pain relieved by anatacids and possible decrease weight. Need uPPER gi endocopy to exclude malignancies, multiple biopsies and repeat after 6-8wks to confirm healing.

163
Q

Gastritis symptoms and RFs

A

•Gastritis: RFs are alcohol, NSAIDs, H.pylori, Reflux/hiatus, atrophic gastritis, granulomas, CMV, ZE, Me-Netriers diaease. Epigastric pain, von. Need upper GI endoscopy if suspicious features.

164
Q

Tests for Peptic ulcer disease

A
  • Invasive - CLO tets, histolgoy, culture
  • Non-invasive - 13C breath test, stool antigen, serology
165
Q

What are the different blood products

A
  • Packed RBCs
  • Platelets
  • FFP - fresh frozen plasma, for clotting
  • Cryoprecipitate 0 made from FFP frozen and thawed multiple times to produce conc clotting factors
  • Plasma derivatives
166
Q

Blood tranafusion basic principles

A
  • Avoid unnecessary/ inappropriate ones. Preventable wrong blood into patient = NEVER EVENTS. ID check, Right tine, right patient, right place.
  • Local policies, lable samples straight away and do one at time
  • Positive patient identification – 3 identifiers. 2 sample rule that 2 different people take samples for blood group.
  • Monitor TPR + BP every 30mins and use dedicated line for transfusion where possible.
  • Transfusion reaction – Fevers, chills, rigors, myalgia, nausea, mouth or throat swelling, hypotension/hypertension,acute bleeding, breatles,s signs anaphylaxis, hypoxia, skin rashes…
  • If ABO incompatible and red cells are transfused – red cell haemolyisis can occur.
167
Q

Rh group and signficiance

A

•Rh group (resus). Most important antigen is RhD. Antibodies to RhD can develop only after someone is exposed to Rhd antigens via transfusion, pregnancy or organ tranplntation. Anti RhD antibodies destroy RhD positive red cells – hemolytic transfusion reaction. In pregnancy, anti D Abs can cross placenta and cause hemolytic disease of the newborn. So RhD negative girls/women of child bearing age should not be transfused with RhDpositive red cells.

168
Q

Blood Compatibility

A
169
Q

Plasma compatibility

A
170
Q

Intepretatign ABGS

A
171
Q

Fluid Mx for those with gI haemorrhage

A
  • Transfusion for patients with massive bleeding. Do not offer platelet transfusion for those not actively bleeding and hemodynamically stable.
  • Offer plaetely transfusion to patients atively bleeding and platelet count of less than 50x10^9/l
  • Offer fresh frozen plasma to patients actively bleeding and prothrombin time greater than 1.5 times normal
  • Offer prothrombin complex concentrate to patients who are taking warfarin and actively bleeding
  • IV Access, fluid resus, transfusion (actively bleeding and hypovalaemia). Generally if <70Hb then aim to increase to 90Hb. 1 unit (bag) increased 10Hb.
  • Hemodynamically unstable: Establish IV access and provide supplemental O2. Treat hypotension with rapid bolus infusions of isotonic crystalloid. Transfusion for severe ongoing bleeding.
172
Q

Management with gI haemorrhage

A
173
Q

DDx of patient with GI bleeding

A

Common:

  • Peptic ulcer disease – surface defect in mucosa of gI tract
  • Oesophageal varices – enlarged veins in oesophagus. Ften occur due to obstructed flow through portal vein due to portal hypertension
  • Oesophagitis – inflammation in lining of oesophagus. Often caused by acid reflex which irritates and damages oesophgaus lining.
  • Mallory Weiss Tear – tear off mucus membrane at gastro oesophageal junction
  • Bulimia
  • Gastric cancer/ malignancy.
  • Patients with chronic bleeding usually present with iron deficiency anaemia. Oral iron to treat this.

Uncommon:

  • Bleeding disorders
  • Portal hypertension
174
Q

Bloods in GI bleed and why?

A
  • FBC – WBC can increase during acute emergencies
  • Hb – drop indicates GI bleed
  • U/Es – urea increases in GI bleed
  • LFTs – Liver diseases and deranged coag factors can cause GI bleed
  • CRP + Blood culture – infective cause
  • Cross match x2 – if need blood transfusion
  • Ferritin, Vit B12 + folate – anaemia
  • Coag – INR, Prothrombin time
175
Q

Glasgow Batch fORD SCORE - WHAT IS THIS

A

Predicts likelihood of needing endoscopy.

  • Malena, BP, HR, Kidney disease, Liver disease, CVS disease, gender, urea, syncope, Hb
  • Score >6 indicates requiring endoscopy
176
Q

Weight loss DDx, and investgations

A

•Initial investigations: Test urine for sugar + protein, FBC, ESR or CRP, fasting blood glucose, U&E, calcium + bone profile, LFT, PSA, TSH, FT4, CXR, US scan abdomen.

DDx: Advanced malignancy, psychiatric ilness, non malignant gI disease, alcholism, undiagnosed/uncontrolled DM, thyrotoxicosis, pulmonary TB, Addison’s disease, acute infections…

177
Q

Cancer cachexia, why

A
  • Cancer cachexia is a wasting syndrome characterized by loss of skeletal muscle and fat. Common in those with metastatic cancer.
  • It is caused by a beak down of muscle tissue and fat.
  • Caused by release of chemokine eg, TNF and loss of appetite associated with cancer
178
Q

Nutritional management approaches to cancer cachexia

A
  • Increase fluid intake
  • Promote high calories protein/fluids throughout the day eg, full fat products
  • limit fluids after meals to avoid feeling full
  • High calories, high protein foods (eg, cheese, eggs, nut butters, avocados…)
  • Oral nutritional supplements as needed to augment diet: particularly if patient has symptoms that interfere with nutritional intake or absorption.
  • Manage contributors to anorexia eg, chronic nausea, constipation, taste alterations
  • Urgent Mx = Refer to oncology dietician. Consider need for enteral/parenteral nutritional support.
179
Q

Haematemesis - what is it, intiial invetsiagtiosn and DDx

A
  • Bright red blood and/or passage of black tarry motions (melaena) Implies bleeding from upper GI tract.
  • Initial investigations : FBC, U&E, LFT, clotting screen, G&S, monitor Hb, possible ph study, poesophagosgastroscopy.
  • DDx - bleeding peptic ulcer, gastric erosion, oesophagela varices, ammloy weiss tea, oesophageal carcinoma, gORD, Hiatus hernia, Ingestion of corrosives, Meckels diverticulum, False haematemesis
180
Q

Causes of Hematuria (blood in urine) and some ix

A
  • Transient – UTI or menstruation
  • UTI – pyleonephrtitis, cystitis
  • Malignancy – Kidney, ureter, bladder
  • Polycystic kidney disease
  • IgA nephropathy, glomerulonephritis

Invetsigations: •Urological assessment, imaging + cystoscopy to rule out renal tract malignancy or calculi indicated if -> VH, sNVH + aNVH +>40

181
Q

Enteric NS and gut motility in normal digestion and absorption

A
  • Anatomy: Ileum contains mucosal folds with villi and microvilli Villus is made of core blood vessels, lymphatics and plasma and lymphocytes
  • Enteric NS – acts independently but can be influenced by autonomic nervous system. Myenteric and submucosal plexuses. Communicates with CNS via autonomic afferent and efferent pathways
  • Gut motility – Contractile movements controlled by enteric NS. Interstitial cells of canal = pacemaker cells. Migrating motor complex occurs to clear gut tube of food and debris before next meal occurs. Motor quincence, irregular contractions and regular phasic contraction occurs.
182
Q

Absroption - what is it

A

•Absorption: Nutrients absorbed into blood across mucosa of small intestine. Water and electrolytes absorbed. Low intracellular Na+ required to permit absorption. Main mechanisms are simple diffusion, facilitated diffusion and active transport.

•Products released at apical membrane and diffuse into enterocyte, products are replaced to form original lipids and packed to form a chylomicron which enter lymphatic system via lacteals.

183
Q

Carbohydrate absorption

A

•Carbs – Main = glucose, galactose + fructose. Salivary amylase, pancreatic amylase (starch -> disaccharide), Brush border enzymes hydrolyze disaccharides. Absorption is SGTL1 allows for absorption of glucose + galactose across apical membrane. They leave cell vi GLUT2 receptors -> basolateral membrane -> blood. Fructose is uptake into cell via GLUT5 via facilitated diffusion

184
Q

Protein absorption

A

•Protein – Pepsis (breaks down proteins -> AA + oligopeptides). Brush border enzymes + pancreatic enzymes for further digestion. Absorption – across basolateral membrane via facilitated diffusion

185
Q

Lipids absorption

A

•Lipids: Lingual + gastric lipases. Bile acids released into duodenum -> emulsification into micelled with large SA. Micelles -> fatty acids, glycerides, cholesterol.

186
Q

Obesity DDx, health risks

A
  • Assess distribution body fat (trunk or limbs), pitting or non pitting (adiposity) oedema. Test urine for protein and glucose, FBC, fasting glucose, U&E, TSH, FT4.
  • DDx: Genetic (Prader Willi syndrome + LM syndrome), Hypothyroidism, PCOS, Cushing’s syndrome/disease, Hypothalamic damage (trauma/tumour which disrupts satiety signals)
  • Leptin deficiency can cause extreme obesity. Hormones like grehlin (stimulates appetitie), leptin, cortisol, T3+4, microbiome and insulin affect weight. Most obese people become insensitive to leptin. When overweight people try loose weight, grehlinincreases and stays like that for while so feel more hungry .
  • Health risks = HBP, High LDL, T2DM, coronary heart disease, stroke, gallbladder disease, breathing problems, cancers, CV disease increased risl

•How to approach

  • Mx - education, specialist programs, surgical (gastric bands- reistrictive vs malabsroptative). Orlistat inhibits pancreatic lipase (so steatorrhoea)., GLP-1 (inhibit appetite centrally)
  • Bariatric surgery: >40 BMI or comorbidity and >35BI, tried non surgery with intesntivecmanagement, committed to life longsurgery.
187
Q

What is fattism

A

•Prejudice or discrimination against people who are fat.

188
Q

Anterior neck lump DDx

A

Anterior neck lump – moving with tongue protrusion and swallowing. Suggests extrathyroid tissue, need US (and possible radioisotope can, CT, histology)

  • Thyroglossal cyst = suggested by fluctuant, cystic lump in midline or just to left.
  • Ectopic thyroid tissue – solid lump in midline or just lateral.
189
Q

Neck lump - goitres

A

Neck lump – Moving with swallowing but not with tongue protrusion. Suggests goitre (or something attached to thyroid gland). TSH/FT4.

  • Thyrotoxic goitre – sweating, fine tremor, tachycardia, weight loss, lid lag. Confirm with increase FT4/3 and D TSH. US, isotope
  • Hypothyroid goitre – cold intolerance, tired, constipation, bradycardia. Increase TSH, dexrease FT4. US, thyroid antibodies (+ve).
  • Euthyroid goitre – Absence sweating, fine tremor, weight change, cold intolerance, tiredness, lid lag, normal bowel habit, normal pulse rare. Normal FT4/TSH
190
Q

Oedema - what is it and the pathology

A
  • Oedema = Abnormal collection of fluid in tissues, which an collect in interstitial or intracellular spaces. Interstitial oedema is either changes in capillary dynamics from increased hydrostatic pressure or decreased plasma oncotic pressure; stimulation of inflammatory immune response or lymphatic system obstruction. NET filtration pressure (hp- OP).
  • Patho = Expansion ECV by at leats 2L, venous tone (determines capacitance of blood and hydrostatin pressure), capillary permeability, oncotic pressure (albumin), lymphatic drainage.
191
Q

Causes of oedema

A
  • HF – reducedtion CO -> RAAS system activation -> increased TPR -> Na+ and water retention -> increased ECG volume -> increased venous pressure -> oedema.
  • Hepatic cirrhosis – increased TPR -> Na+ and water retention -> oedema. NoO production causes vasodilation
  • Nephortic syndrome
  • Sodium retention (drug induced, oestrogen, NSAIDs)
  • Drug induced (Eg, Ca2+ channel blcokers)
  • Increased capillary permeability to proteins.
192
Q

What doe sbilateral oedema suggest

A

Bilateral oedema -> systemic disease with increased hydrostatic pressure or reduced intravascular oncotic pressure. RHF, Decreased albumin, venous insufficiency, malabsorption, nutritional (High Na+/low albumin/malabsorption disease)

193
Q

Invetsiagtions for Oedema

A

•Ix = Xray, US, MRI, bloods, urine analysis

194
Q

Pitting, Non pitting and periorbital oedema

A
  • Pitting Oedema – Fluid can be squeeze out of veins (increase hydrostatic pressure eg, DVT, RHF) or diffuse out because of reduced oncotic pressure (low plasma proteins eg, cirrhosis, nephrotic syndrome, protein loosing enteropathy_ leading to osmotic gradient with tissues. Treat the problem, ea tless salt, diuretics, wear compression things.
  • Non-pitting oedema – Non identifiable is lymphoma due to poor lymphatic drainage, Ca be due to radiotherapy, malignant infiltration, infection, filariasis or rarely primary lymphoedema. Treat underlying. Mild lymphedema can resolve with exercise as enhance lymphatic flow.
  • Periorbital oedema – Oedema around face – eyelid skin is very thin so periorbital oedema usually first sign (eg contact dermatitis), angioedema (hereditary, infection. If proptosis, think graves disease, Ct diseases (eg, SLE, sarcoid, amyloid). Assess for systemic disease before putting down to allergies. Underlying cause.
195
Q

CT colongraphy

A

•uses gas or air to inflate colon and then CT. Screens for polyps or cancer of large intestine. Minimally invasive, low risk of eprforaitonof colon, when increased risk for colonoscopy, elderly, cat detect abnormalities outside colon. Well tolerated. Risk injury or perforation from air, cancer form exposure to radiation.

196
Q

Colonscopy

A

•– Offered to hose with iBD whose symptoms started >10yrs ago + determine risk of developing CRC. Tissue biopsy, detect and remove polyps (can become cancerous), detect bowel cancer, CD/UC. Risks are heavy bleeding, tears in colon, inflammatory or infection of pouches (diverticulitis)… Abdo discomfort

197
Q

Barium meal

A

•Ba Meal – Xray fluoroscopy used in real time, images taken as swallow barium ad it passes into stomach and small bowel. Diagnose things like cancer, hiatus hernia, structural dysfunction (diverticula, polyps), varicose, GERD, ulcers, achalasia. Give info on swallowing action, ulcers, growths etc. May feel bloated after, constipation for few days.

198
Q

MRI

A

•non invasive, no radiation, takes a while, not immediately available usually

199
Q

What is haematopoiesis, early and adult one.

A
  • Haematopoiesis = formation and development blood cellular components. All cellular ccomponents derived from haematopoietic stem cells. Predominate site is bone marro win adults (medulla) where they can go to myeloid and lymphoid cell lines.
  • Early Haematopoiesis – begins in embryonic yolk sac in utero at around 14-19 days. In foetus then mostly in liver in 2nd trimester. From 7m BM takes over. First 4years life then almost all marrow cavities have (haematopoietic tissue) red marrow. Which goes to axial skeleton and long bones As we age this decreases in quantity and replaced with fat (yellow marrow).
  • Adult haematopoiesis – BM, if defective or insuffieinc then expansion ed marrow might occur. Haematopoiesis also can occur in liver and spleen (extramedullary).
200
Q

How to regulate haematopoiesis

A

•Regulation – deoendent on glycoprotein GFs. These dirve proliferation + differentiation of progenitor cells. EPO proliferation + maturation RBCs, released by kdineys), TPO (produced by liver and essential for platelet porductin), intelruekins, G-CSF

201
Q

Components of blood

A

•– specilaised bdy fluid, essential transport medium. 4 main components (55% plasma, 45%red cells, WBC, platelets) buffy coat <1%

202
Q

Plasma components

A
  • Plasma – 90% water, trnasports nutrients and waste through body. Salts, lipids, hormones, proteins dissolved in it. Proteins (albumin, immunoglobulins, clotting factors and fibrinogen).
  • Albumin prevents blood form loosing too much water and consistency as travels. Travels various blood components and nutrients.
  • Immunoglubins are Abs which are essential in fighting pathogens with white cells
  • Clotting factors – in combo with platelets, control haemorrhaging.
  • RBC, WBC, platelets are suspended in plasma.
  • Buffy coat – contains
203
Q

What are red cells

A

•erythrocytes, densest component, haematocrit, non nucleated, essential for 02 transfer around body and removal of CO2. Biconcave, around 120days lifespan then removed by splenic + liver macropahges. 3 main component – red clelmembrane, metabolic machinery, Hb. Generation of erythrocytes in BM and depend on EPO release.

204
Q

Haemaglobin structure

A

•Hb – 4 polypeptide globin chain, each with haem group. In a adults, 2 alpha, 2 beta chains

205
Q

Leucocytes what are they

A

•Leucocytes – Nucleated cells in blood and reticuloendothelial system. Important in immune system, protects against pathogens. Also immunomodulation, immune surveillance and inflammation.

206
Q

Neutrophil role

A

•Neutrophils – granulocytes, 70% circulating WBC, ingets pathogens + attacked with enzymes

207
Q

Monocyte role

A

•Monocytes – largest leucocyte, able to enter peripheral tissue when become macrophages. Able to phagocytose pathogens.

208
Q

Basophil role

A

•Basophils – large granules with histamine.

209
Q

Eosinophil role

A

•Eosinophils – defense against parasitic organisms and allergic conditions.

210
Q

Lymphocytes

A

•Lymphocytes – Essential part of adaptive immune response. T lymphocytes (cell mediated immunity), b lymphocytes (humoral immune response + production immunoglobulins). Agranulocytes. Variable life span

211
Q

Platelets

A

•Platelets – Critical in formation of haemostatic plug at site of vascular injury. 1/5th size of erythrocytes and lakc nucleus. Derived from megakaryocytes and lack nucleus. Large numbers platelets (released directly into BM sinusoids form single megakaryocytes). Life span 10-12days.

212
Q

Cytopaenias

A

•Cytopenia = reduction in cellular components of the blood. Low Hb is anaemia, low lecucoytes is leucopenia/neutropenia/lymphopenia. Low platelets is thrombocytopaenia.

  • Anaemia = Fatigue, lethargy, pallor, shortness of breath, chest pain
  • Leucopaenia = neutropaenia, increased risk infections (bacterial/fungal)
  • Thrombocytopaenia = petechial rash, bruising/pupura/ecchymoses, blood blisters, bleeding
  • Pancytopaenia = Combo anaemia, leucopaenia and thrombocytopaenia. Caused by
  • Failure of production of blood cells (bone marrow infiltration due to blood cancer or met cancers, aplastic anaemia-red of haempoietic stem cells) and vitamin B12 and folate def.
  • Ineffective haematopoiesis – myelodysplastic syndrome
  • Increased destruction of blood cells – Hypersplenism, autoimmune disorders, paroxysmal nocturnal haemoglobinuria (rare), Myelosuppression after irradiation or medication (chemo)
213
Q

Haematology tests that we use

A
  • FBC (purple) – Hb conc, leucocytes (differential), platelets, MCV (assess anaemia). Will detect anaemia, neutropenia, neutrophilia, lymphocytosis, thrombocytopaenia, thrombocytosis (inflammation/infection/myeloproliferative disorders)
  • Blood smear (blood film) – thin layer blood smeared on glass microscope slid. Stained and allows blood cells to be examined unde microscope.
  • Bone marrow biopsy – sample to be examined under mcirosocpe. Boen marrow aspirate, bone marrow trephine.
  • Coagulation – Measures ability to clot and how long it takes. Can assess risk of excessive bleeding.
214
Q

Haemostasis - describe and the ingredients

A

Haemostasis:

  • Platelet plug – formed in response to vascular injury
  • Clotting cascade – different pathways intrinsic/extrinsic/common pathway), coagulation factors
  • Ingredients – intact vessel wall, coag factors (factor def, inhibitors), platelets (abnormal number, abnormal function).
215
Q

What does a cogaulation screen involve

A
  • Prothrombin time (PT) – time for blood to clot via extrinsic pathway
  • INR (anticoagulant)
  • APTT- activated partial thromboplastin time – tiem take for blood to clot via intrinsic pathway
  • Fibrinogen
216
Q

What do ask in a bleeding hsitory

A

Exact nature, size, site, time stop, spontaneous, surgical challenges previously, menstruation, FH, drug history, timing and results.

217
Q

Haemophilia A and B- msising factors

A

Hameophilia A (FVIII def)

Haemophilia B (FIX def)

218
Q

What is lymphoproliferative disease

A

Lymphoproliferative disease = Dysregulated control of lymphocyte proliferation

219
Q
A
220
Q

Describe common pregentations of Lymphoprolfierative disorders

A
  • Weight loss
  • Anaemia – bone marrow infiltration with cancer cells -> minimal room for normal cell types
  • Cytopaenias
  • Lymphadenopathy
  • bruising/bleeding easily
  • Sweating
  • Fatigue/lethargy
  • Can lead to lymphocytosis, lymphadenopathy + involvement to extranodal sites eg, bone marrow
221
Q

DDx and invetsiagtions of Lymphoprolfierative disorders

A
  • Malignant -> Lymphoma or Leukaemia
  • Non-malignant -> Result from infection eg, EBV
  • Lymphadenopathy
  • Reactive -> SLE -> Lymphadenopathy

Investigations:

  • FBC
  • Serological testing for cytomegalovirus + EBV (can cause LP disease)
  • TB testing via AFB
  • ESR
  • Peripheral blood smear
  • Bone marrow biopsy and aspirate
222
Q

What is Acute lymphoblastic leukaemia and main symptoms, Tx

A

•ALL = malignancy of lymphoid cells, affecting T/B lymphocytes cell lineages, arresting maturation and promoting uncontrolled proliferation of immature blast cells, with marrow failure and tissue infiltration. Ionizing radiation during pregnancy, and downs syndrome are important associations. Commonest cancer childhood, rare in adults, CNS involvement common

Marrow failure (decrease Hb), infection (decrease WCC), bleeding (decrease platelets). Hepato/spleno meg

223
Q

Acute myeloid leukaemia - what is it, signs/s

A
  • Neoplastic proliferation of blast cells derived from marrow myeloid elements. Progressives rapidly. WHO classification based on histological, cytogenetics + molecular genetics.
  • Signs + Symptoms = Marrow failure (Anaemia, infection, bleeding, DIC in promyelocytic anaemia), infiltration (hepato/spleno-megaly, gum hypertrophu, skin involvement, CNS rare.
224
Q

Myelodysplastic syndromes

A
  • Heterogenous group of disorders that manifest as marrow failure with risk of life threatening infection and bleeding (med survival 6m-6y). Mostly primary but can develop secondary to chemo or RT. 30% transform to acute leukaemia.
  • Tests = Pancytopenia with decreased reticulocyte count. Marrow cellularity is usually increased due to ineffective haematopoeisis. Ring sideroblasts may also be seen in marrow.
  • Tx = Multiple transfusions of red cells or platelets as needed. Erythropoietin +/- G-CSF. Allogenic stem cell transplant option. Low intensity treatment for quality of life.
225
Q

Chronic lymphocytic leukaemia CLL

A
  • Commonest leukaemia. Progressive accumulation of malignant clone of functionally incompetent B cells. Mutations, trisomies, deletions influence risk. Mostl over 70, median diagnosis 65.
  • Symptoms = often none, presenting as surprise finding out routine FBC. May be anemia or infection prone, or have decreased weight, sweats, anorexia if severe. Autoimmune complciations – ITP, AIHA
  • Sings = Enlarged, rubbery, non-tender nodes. Splenomegaly, hepatomegaly.
226
Q

Hodgkins lymphoma

A
  • Lymphomas are disorders caused by malignant proliferations of lymphocytes. Accumulate in lymph nodes causing lymphadenopathy but may also be in peripheral blood or infiltrate organs.
  • Incidence: Young adults, elderly. RFs are affected sibling, EBV, SLE, post transplant.
  • Symptoms: Enlarged, non-tender, ‘rubbery’ superficial lymph nodes, size may fluctuate and become matted. Some have fever, weight loss, night sweats, pruritus’, lethargy and may be alcohol induced lymph node pain. Mediastinal lymph node involvement can cause mass effect or direct invasion.
  • Signs: Lymphadenopathy, cachexia, anaemia, spleno/hepato-megaly.
227
Q

Non-Hodgkins lmyphoma

A
  • All lymphomas without Reed-Sternberg cells. Most derived from B cell lines, diffuse elarge B cell lymphoma is commonest. Not all centre nodes.
  • Causes – Immunodeficiency (drugs, HIV, HTLV-1, H.pylori, toxins, congenital
  • S/S = Superficial lymphadenopathy, extranodal disease (Gut- Gastric MALT, Non-MALT gastric lymphomas, small bowel lymphomas, Skin, oropharynx, bone, CNS, lung), systemic features (fever, night sweats, weight loss), pancytopenia from marrow involvement (anaemia, infection, bleeding (decrease platelet).
228
Q

Myeloproliferative disorders and classifications

A
  • Caused by clonal proliferation of haematopoietic myeloid stem cells in bone marrow. These cells retrain ability to differentiate into RBCs, WBCs, or platelets, causing excess o one or more of these cell types.
  • Classification:
  • RBC -> Polycythaemia Vera
  • WBC -> Chronic myeloid leukaemia
  • Platelets -> Essential thrombocythaemia
  • Fibroblasts -> myelofibrosis
229
Q

Polycythaemia (MPD)

A
  • Polycythaemia, also known as erythrocytosis, means having a high concentration of red blood cells in your blood. This makes the blood thicker and less able to travel through blood vessels and organs.
  • Relative Polycythaemia
  • Absoloute polycythaemia –
  • Primary polycythaemia
  • Secondary Polycythemia
  • Causes polycythaemia = Smoking, copd, increased epo production by kidney, renal artery stenosis, renal tumours.
230
Q

Polycythaemia vera

A
  • Persistent raised Haematocrit males >0.52, females >0.48.
  • Malignant proliferation of a clone derived from one pluripotent stem cell. A mutation In JAK2 present in most (primary causes, JAK2 V617F mutation positive). The eyrthroid progenitor offspring are unusual in not needing erythropoietin to avoid apoptosis. There is excess proliferation of RBCs, WBCs, and platelets, leading to hyperviscosity and thrombosis. Commoner in >60
  • Presentation: May be asymptomatic and detected on FBC or present with vague symptoms due to hyperviscosity: headaches, dizziness, tinnitus, visual disturbance. Itching after hot bath, erythromelalgia, burning sensation in fingers and toes.
  • Signs = facial plethora + splenomegaly. Gout may occur due to increase urate from RBC turnover. Features aterial or venous thrombosis may occur.
231
Q

Thrombocythaemia

A
  • Primary causes + secondary causes
  • Essential Thrombocythaemia: myeloproliferative neoplasms, linked to JAK2 plus othe rmutations (CALR/MPL). Median age diagnosis 60years.
  • Clonal proliferation of megakaryotcytes leads to persistently increased platelets with abnormal function causing bleeding or arterial and venous thrombosis and microvascular occlusion (headache, atypical chest pain, light headed, erythromelagia. Exclude other causes thrombocytosis. Can have hemorrhagic symptoms and thrombosis and splenomegaly.
  • Treatment = aspirin 75mg only when low risk under 60 and olt count <1500 and lw symptom burden. Otherwise add hydroxucarbamide, anagrelide, interferon.
232
Q

Myeloma (plasma cell dyscrasia PCD)

A
  • PCDs – abnormal proliferation of single close of plasma or lymphoplasmacytic cells leading yo secretion of immunoglobulin Ig or fragment causing dysfunction of many organs (esp kidney). The Ig is seen as monoclonal band or paraprotein, on serum or urine electrophoresis.
  • Clinical: osteolytic bone lesions, hypercalcaemia, anaemia, neutropenia, thrombocytopenia, recurrent bacterial ifnections,r enail impairment.
  • Tests: bloods (normocytic, normochromic anaemia), film (peristsently increased ESR). Urea and creatinine up, Calcium up. Bone marrow biopsy, screening test, imaging (xrays for lytic punched out lesions/ vertebral fractures etc).
233
Q

Myelofibrosis

A
  • Hyperplasia of megakaryocytes which produce platelet derived growth factor, leading to intense marrow fibrosis and haematopoiesis in the spleen and liver (massive hepatosplenomegaly).
  • Presentation: hypermetabolic symptoms- night sweats, fever, weight loss, abdo discomfort, bone marrow failure (decrease Hb, ifnections, bleeding)

Teardrop RBCs

234
Q

Chronic myeloid leukaemia

A
  • CML = uncontrolled clonal proliferation of myeloid cells. Myeloproliferative disorder having common features like splenomegaly. Mostly 40-60years, male predominance, rare in childhood.
  • Philadelphia chromosome present in most – t(9:22) forming fusion gene BCR/ABL on C22 with tyrosine kinase activity. Worse prognosis
  • S/S= mostly chronic and insidious: decrease weight, tired, fever, sweat, features gout, bleeding, abdo discomfort (splenomegaly), hepatomegaly, anemia, bruising.
  • 3 phases = chronic (few months/years), accelerated (mor symptoms), blast transformation (acute leukaemia features with possibly death)

Imantinib

235
Q

Vomiting DDx

A
  • Mechanism – Neuro is stimulation of area postrema initiating emesis reflex in response to noxious agents. Peripheral is diseases in PNS
  • Gastritis, GORD, Peptic ulcer disease, Acute gastro enteritis, acute coronary syndrome, menderes disease, acoustic neuroa.
236
Q

Abdominal pain in relation to area of abdomen

A
237
Q

Tiredness DDx + IX

A
  • Ix – FBC, U&E, fasting blood glucose, TSH, FT4 and other
  • Depression – early morning wking, fatigue wors eon morning, never goes
  • Anaemia – pale
  • Primary hypothyroidism – cold intolerance, tired, constipation, bradycardia
  • Sleep apnea syndrome = frequent awaking at night, snoring, breathing pauses, sleepiness in day
  • Drug-induced – sedating drug or anti-epileptic
  • Post-viral fatigue – history of recent viral illness, esp glandular fever
  • T2DM – thirs,t polyuria, poldispa, FH
  • Chronic fatigue syndrome – imapire dmemory/conc, unexplained dmuscle pain, polyarthralgia, nrefreshing sleep, post exertional malaise over 24h, persistent sore throat, unexplained tender cervical or axillary nodes
  • Poor sleep habit – long working horus etc
  • Parasomnias – cataplexy, narcolepsy, daytime somnolence.
  • Metabolic disease – Diabetes, thyroid.
238
Q

Omprezaole

A
239
Q

Randitidine

A
240
Q

Ibuprofen

A
241
Q

Steorids MOA

A
242
Q

Aspirin

A
243
Q

Clopidogrel

A
244
Q

Warfarin

A
245
Q

Metformin

A
246
Q

Gliclazide

A
247
Q

Ramipril

A
248
Q

Amlodipine

A
249
Q

Simvastatin

A
250
Q

Furosemide

A
251
Q

Angiotensin II receptor blockers

A
252
Q

Spironolactone

A
253
Q

Thyroxine

A
254
Q

Causes of abdominal distention

A
  • Distention = substances such as air (gas) or fluid, accumulate in abdomen causing its expansion
  • Causes of Abdominal distention (6Fs)
  • Fat
  • Fluid (ascites/oedema)
  • Flatus (obstruction/ileus)
  • Faeces
  • Fetus
  • Flipping big tumour
255
Q

Abdo Exam

A
256
Q

INTERPRETING THE IRON PANEL

A
257
Q

Interpreting FBCs

A
258
Q

How to interpret coag tests

A
  • Prothrombin time – Measure Vitamin K dependent extrinsic pathway.
  • Fibrinogen – evaluates that part of the clotting process in which soluble fibrinogen is converted to fibrin threads to form a clot
  • APTT – Activated partial thromboplastin time – Measures the intrinsic coag pathway.
  • D Dimer – clot degradation product
  • INR <1.1 – high INR means blood clots more slowly than desired, and a lower INR means your blood clots more quickly than desired.
  • Thrombin time – Measures time it takes for fibrinogen to form fibrin
259
Q

Examine the neck

A
260
Q

Intepret basic endocrinology tests

A
261
Q

U&E interpretatin and causes

A
262
Q

Basic exam of fluid status

A
263
Q

Examine for signs of signficiant disease eg, BD and cancer

A

Cancer: fatigue, unwell, unexplained weight los, anorexia/dysphagia, vomiting, nausea, swellings or lumps

IBD:

  • Crohn’s: bowel ulceration, abdominal tenderness, perianal abscess, anal strictures, clubbing, skin, joint + eye problems
  • UC: Acute -> fever, tachycardia, tender/distended abdomen or chronic signs -> clubbing, apthousoral ulcers, erythema nodusum, episcleritis, conjunctivitis, arthritis, nutritional deficiencies.
264
Q

Perform a basic assessment of nutritional status with reference to both underweight and overweight individuals

A
  • Antropometry: Assess component’s of body and body composition: Weight and % weight change, BMI, Skin fold thickness, upper arm circumference.
  • Biochemistry: Hib, albumin, CRP, WBC, HbA1c, Na+, urea, Ca2+ and phosphate, Mg2+, micronutrients
  • Clincial: disease state of patient can increase risk of malnutrition
  • Eg: Cancer, COPD, CHF, Neuro disorders etc. Symptoms of patient can also impact nutritional status like altered bowel habits.
  • Dietary: Estimate BMR, Determine caloric and fluid requirements, dietary assessment (total calories and overall quality diet)
  • Environment – social and physical factpr
  • Assess risk of refeeding syndrome.